BIO 365: Final Practice Questions

Pataasin ang iyong marka sa homework at exams ngayon gamit ang Quizwiz!

You are called on to do a venous catheterization to measure central venous pressure in the heart. Which muscle of the neck would provide the best area of approach to the internal jugular vein? Anterior scalene Digastric Omohyoid Sternocleidomastoid Sternohyoid

Sternocleidomastoid The internal jugular vein is found in the carotid triangle, which is bounded medially by omohyoid, laterally by sternocleidomastoid, and superiorly by the posterior belly of digastric. Of these three muscles, sternocleidomastoid is the easiest landmark to find on the neck and the most practical muscle to find when looking for the carotid triangle and the internal jugular vein. The anterior scalene is posterior to the internal jugular vein, and is not related to the vessel. Sternohyoid is a strap muscle in the muscular triangle; it is not associated with the internal jugular vein.

The difficulty in swallowing was due to involvement of which muscle that elevates the tongue? Genioglossus Hyoglossus Styloglossus Stylohyoid Stylopharyngeus

Styloglossus Styloglossus retracts and elevates the tongue. Genioglossus is a large, fan shaped muscle with many actions: its inferior fibers protrude the tongue, its middle fibers depress the tongue, and its superior fibers draw the tip back and down. Hyoglossus retracts and depresses the tongue. All of these muscles are innervated by the hypoglossal nerve (CN XII). Stylohyoid elevates and retracts the hyoid bone. It is innervated by the facial nerve (CN VII). Stylopharyngeus elevates the larynx--it is innervated by the glossopharyngeal nerve (IX).

What are the important associations with the other walls of the ear?

The lateral wall is formed by the tympanic membrane, and the chorda tympani courses across this membrane. The anterior wall contains the opening of the auditory tube and a semicanal for tensor tympani. The posterior wall of the ear has the aditus ad antrum, which is the entrance to the mastoid antrum and air cells. The facial nerve (VII) and a small branch of the facial nerve which innervates stapedius are found on the posterior wall.

During a hysterectomy, care must be taken in ligation of the uterine vessels because they cross the _________ superiorly. ureter round ligament of the uterus ovarian artery lumbosacral trunk inferior hypogastric plexus

Ureter The uterine vessels cross over the ureter as the ureters pass through the base of the mesometrium. Remember--the ureters must travel through the mesometrium to reach the base of the bladder. The relationship between the ureter and the uterine vessels is very important--you should remember this!

Two nerves usually emerge from between the two heads of the lateral pterygoid muscle: the anterior deep temporal nerve and the: Masseteric n. Buccal n. Lingual n. Inferior alveolar n.

buccal The lateral pterygoid muscle has 2 heads: The superior head of the muscle inserts into the disc and capsule of the temporomandibular joint while the inferior head inserts into the neck of the mandible (the pterygoid fovea). There are two nerves coming between the heads of the lateral pterygoid muscle: the anterior deep temporal nerve and the buccal nerve.

The triangle in which the superior branch of the ansa cervicalis separates from the hypoglossal nerve is the: Carotid Muscular Subclavian Submental Occipital

carotid The superior branch of ansa cervicalis runs with the hypoglossal nerve until the carotid triangle. In this triangle, the superior branch of ansa cervicalis separates from the hypoglossal nerve and runs in the carotid sheath, anterior to the carotid artery. Remember--the carotid sheath is found in the carotid triangle, a space bounded laterally by the sternocleidomastoid muscle, superiorly by the posterior belly of the digastric, and anteriorly by the superior belly of omohyoid. The other triangles listed are not the spaces where ansa cervicalis separates from the hypoglossal nerve.

The artery which supplies blood to the major erectile body in both the male and female is the: Artery of the bulb Dorsal artery of the penis/clitoris Deep artery of the penis/clitoris Posterior labial/scrotal artery Superficial external pudendal artery

deep artery of the penis/clitoris The deep artery supplies the corpus cavernosum of the penis/clitoris, which is the major erectile body. It is one of the two terminal branches of the internal pudendal artery, with the other one being the dorsal artery of the penis/clitoris. This artery supplies superficial structures. The artery of the bulb supplies blood to the bulb of the penis and the bulb of the vestibule. Although the bulbs are erectile tissue, the corpus cavernosum is the main erectile body. The posterior labial/scrotal artery supplies exactly what you would guess--the posterior labia or scrotum. It is a branch of the perineal artery. Finally, the superficial external pudendal artery supplies the skin and superficial fascia of the upper medial thigh, as well as the skin of the pubic region. It is a superficial branch of the femoral artery.

Which structure is attached to the center of the tympanic membrane? Foot plate of the stapes Handle (manubrium) of the malleus Long process of the incus Tragus Utricle

handle (manubrium) of the malleus The handle of the malleus is attached to the center of the tympanic membrane. The head of the malleus then articulates with the body of incus, and the long process of incus articulates with the head of stapes. The foot plate of stapes sits in the oval window. Take a look at Netter Plate 88 for a better idea of how these ossicles connect!

Which nerve is endangered during surgical removal of an impacted third mandibular molar tooth? Hypoglossal n. Glossopharyngeal n. Inferior alveolar n. Lingual n.

inferior alveolar nerve The inferior alveolar nerve provides sensory innervation to the mandibular teeth. This nerve runs in the mandibular foramen, near the roots of the teeth. So, it might be endangered when removing an impacted tooth. The hypoglossal nerve travels lateral to the carotid vessels and through the floor of the mouth. The glossopharyngeal nerve consists of pharyngeal branches that provide sensory innervation to the upper pharynx and lingual branches that sweep around the stylopharyngeus muscle and continue to the base of the tongue. Finally, the lingual nerve, which is from the mandibular division of the trigeminal nerve, is found in the floor of the mouth. These three nerves are far from the area of the impacted tooth, so they would not be endangered by the dental procedure.

The nasolacrimal duct empties into which part of the nasal cavity? Hiatus semilunaris Inferior meatus Middle meatus Sphenoethmoidal recess Vestibule

inferior meatus The inferior meatus receives the nasolacrimal duct which conducts tears from the orbit. The semilunar hiatus is found in the middle meatus--it receives drainage from the frontonasal duct (draining the frontal sinus), the anterior ethmoidal air cells, and the maxillary sinus. The sphenoethmoid recess is the opening for the sphenoethmoidal sinus. The vestibule of the nose is the opening of the nose that is covered with skin and stiff hairs.

In acute inflammation (arthritis) of the TMJ, the muscle most likely to be affected by the inflammatory process is the: Temporal Medial pterygoid Masseter Lateral pterygoid

lateral pterygoid The inferior head of the lateral pterygoid inserts into the neck of the mandible, while the superior head of the lateral pterygoid inserts directly into the capsule and articular disk of the temporomandibular joint. So, this muscle would be affected by arthritis of the joint. This muscle is the only one that opens the mandible, so this movement might be weakened if there was inflammation at the TMJ. Temporalis elevates and retracts the mandible; it inserts on the coronoid process of the mandible and the anterior surface of the ramus of the mandible. The medial pterygoid muscle protracts and elevates the mandible; it inserts on the medial surface of the ramus and angle of the mandible. Finally, the masseter is the muscle that powerfully elevates the mandible--it inserts on the lower half of the ramus of the mandible. None of these other muscles are inserting into the TMJ, so they would not be impaired quite as much as the lateral pterygoid.

Forward movement of the condyle of the mandible during wide opening of the jaws is accomplished mainly by the: anterior part of temporalis muscle lateral pterygoid muscle masseter muscle medial pterygoid muscle posterior part of the temporalis muscle

lateral pterygoid muscle Remember--the lateral pterygoid muscle is the one muscle of mastication that is responsible for opening the jaw! It also draws the mandible forward. The anterior part of the temporalis muscle elevates the mandible, while the posterior part of temporalis retracts the mandible. Masseter is a powerful chewing muscle that elevates the mandible. The medial pterygoid muscle elevates the mandible and can help to move the mandible forward. But, when the mouth is open, the lateral pterygoid will be the most important muscle for drawing the mandible forward.

The outermost layer of the optic nerve sheath is a continuation of the: Arachnoid membrane Meningeal dura Periosteal dura Pia mater Retina

meningeal dura The optic nerve comes off the base of the brain and passes through the optic canal. As it leaves the brain, it still retains all of the meningeal layer coverings. So, it is covered by meningeal dura, arachnoid membrane, and pia mater. This is significant, because an increase in intracranial pressure will increase the pressure in the subarachnoid space. This may squeeze the optic nerve and make the optic nerve bulge into the eye, a condition known as papilledema. The periosteal dura is the layer of periosteum covering the internal surface of the calvaria. The retina is the inner layer of the eyeball which receives and absorbs visual light rays.

The muscle responsible for raising the floor of the mouth in the early stages of swallowing is the: genioglossus geniohyoid hyoglossus mylohyoid palatoglossus

mylohyoid The mylohyoid muscle elevates the hyoid bone and the tongue and depresses the mandible. It is a muscle in the floor of the mouth that helps elevate the floor of the mouth, so this is the correct answer. The genioglossus is a large, fan shaped tongue muscle that protrudes tongue with its inferior fibers and depresses the tongue with its middle fibers. Geniohyoid is a small muscle in the floor of the mouth that elevates the hyoid and depresses the mandible. Hyoglossus is a tongue muscle that depresses the sides of the tongue and retracts the tongue. Finally, palatoglossus is a palate muscle, innervated by the vagus nerve. It elevates and retracts the tongue.

After the mandibular condyle is moved forward onto the articular eminence (e.g., by opening the mouth widely), what muscle can then retract the mandible? Superficial head of masseter m. Deep head of masseter m. Posterior part of temporalis m. Anterior part of temporalis m.

posterior part of temporalis The fibers of the posterior part of temporalis retract the mandible; the fibers from the anterior part of temporalis elevate the mandible. The anterior and deep heads of the masseter muscle are both important for elevating the mandible--remember, the masseter is the very powerful chewing muscle!

A structure which takes the form of a hood anterosuperior to the clitoris: Frenulum of the clitoris Labia majora Labia minora Prepuce

prepuce The prepuce is a fold of smooth skin that extends over the glans clitoris. It is formed by the joining of the anterior divisions of the labia minora. The frenulum of the clitoris is a small fold found posterior to the clitoris. It is formed by the joining of deeper, posterior, divisions of the labia minora. The labia majora are fat-filled elevations of skin lying on each side of the vestibule of the vagina. The labia minora are smaller folds of skin lying medial to the labia majora. They extend posteriorly and inferiorly from the clitoris.

The most inferior extent of the peritoneal cavity in the female is the: Pararectal fossa Paravesical fossa Rectouterine pouch Rectovesical pouch Vesicouterine pouch

rectouterine pouch Remember: The rectouterine and vesicouterine pouches are the two pouches created by draping the peritoneum over the pelvic organs. These pouches are the two lowest extents of the peritoneal cavity, so to answer this question, you just need to decide which one goes lower. Since the uterus is folded over the bladder, the rectouterine pouch can extend to a slightly lower level than the vesicouterine pouch, which makes C the correct answer. The pararectal fossa is formed by lateral reflections of perineum over the superior one third of the rectum, this space gives the rectum room to fill with feces. The paravesicular fossa is a space near the bladder that allows the bladder to expand. Why is the rectovesicular pouch incorrect? It's only found in males, not females! (But, if the question had asked about males, the rectovesicular pouch would have been the correct answer.)

The predominant muscle most associated with retraction of the mandible is the: lateral pterygoid masseter medial pterygoid temporalis mylohyoid

temporalis Temporalis is the one muscle responsible for retracting the mandible--it pulls the mandible backwards. The lateral pterygoid protracts the mandible, or pulls it forward. It is the one jaw muscle that allows for opening the mouth. The masseter is a powerful chewing muscle that elevates the mandible. The medial pterygoid also elevates the mandible; it has a similar position and action to the masseter, but the ramus of the mandible separates the two muscles. Mylohyoid elevates the hyoid bone and the tongue and depresses the mandible.

The temporomandibular joint is characterized by all EXCEPT: A capsule strengthened by ligaments on its lateral side only A completely flat surface for its gliding action An articular disc Extracapsular ligaments Two joint cavities of different shapes

A completely flat surface for its gliding action The TMJ joint is a synovial joint with two articular cavities. Each cavity is responsible for a different movement at the joint. The lower part of the joint is the hinge component of the joint. When the joint moves, this hinge component of the joint initiates mandibular opening. The upper part of the joint is the gliding component. During joint movement, this gliding cavity moves to terminate mandibular opening. The gliding cavity is the space between the articular disc and the mandibular fossa and articular eminence of the temporal bone--it's not a completely flat surface. There are extracapsular ligaments around the TMJ joint capsule, but these ligaments are on the lateral side only. The lateral ligament reinforces the lateral part of the capsule, while other ligaments (the stylomandibular and sphenomandibular) only have a minor role in stabilizing the joint capsule. There is also an articular disc dividing the two components of the joint, and the two cavities are different shapes. See Netter Plate 11 for a picture of all of these structures.

Which cranial nerve is NOT involved in taste? A. Facial nerve (VII). B. Glossopharyngeal nerve (IX). C. Vagus nerve (X). D. Hypoglossal nerve (XII). E. All of these are involved in taste.

D. Hypoglossal nerve (XII).

An inability to depress the left eye when the eye is fully adducted indicates which nerve is likely damaged? A. Left abducens nerve. B. Right trochlear nerve. C. Left oculomotor nerve. D. Left trochlear nerve. E. Right oculomotor nerve.

D. Left trochlear nerve.

A man displays decreased blood flow to his right face, with an absent arterial pulse in the occipital region, the temporal region, the area behind behind his ear, and the deep structures of the right face. The blood flow to his tongue is unaffected. Based on the symptoms and vessels receiving decreased blood flow, where is the occlusion on the external carotid artery? A. Just below the facial artery. B. Superior to the maxillary artery. C. At the level of the occipital artery. D. At the bifurcation of the common carotid artery. E. At the level of the ascending pharyngeal artery.

A. Just below the facial artery.

Numbness in the anterior aspect of the tongue indicate which of the following nerve is likely damaged? A. Lingual nerve (V3). B. Greater petrosal nerve (VII). C. Hypoglossal nerve (XII). D. Chorda tympani (VII). E. Glossopharyngeal nerve (IX).

A. Lingual nerve (V3).

A person develops a cavernous sinus thrombosis. Because of its relationship to the sinus, which cranial nerve might be affected? Abducens Facial Mandibular V3 Olfactory Optic

Abducens (CN VI) The cavernous sinus is a venous sinus of the brain, lateral to the body of the sphenoid bone. All of the cranial nerves and vessels that pass out of the skull at the superior orbital fissure pass through the cavernous sinus. This includes the abducens nerve, the oculomotor nerve, the trochlear nerve, and the ophthalmic division of the trigeminal nerve. Any of these nerves might be affected by a cavernous venous sinus thrombosis. The facial nerve is not associated with the cavernous sinus. It passes through the internal acoustic meatus and exits the skull through the stylomastoid foramen. The mandibular division of the trigeminal nerve (V3) exits the skull through foramen ovale. The olfactory nerves enter the skull through the cribriform plate of the ethmoid bone. The optic nerve exits the skull through the optic canal. None of these nerves are associated with the cavernous sinus.

An abscess was surgically removed from the middle of the posterior triangle on the right side. During recovery the patient noticed that her shoulder drooped and she could no longer raise her right hand above her head to brush her hair. Which nerve has been cut? Accessory (XI) Ansa cervicalis Facial (VII) Hypoglossal (XII) Suprascapular

Accessory nerve Remember: if the tip of the shoulder is drooping, trapezius is denervated, and the accessory nerve has been damaged! The accessory nerve (CN XI) runs through the posterior triangle, so it is likely to be injured in posterior neck triangle operations. If the accessory nerve is damaged there, the trapezius will be denervated, but the sternocleidomastoid will be OK. Ansa cervicalis is a structure of the cervical plexus that innervates the strap muscles; if this structure was injured, you would note weakness in the strap muscles, not the trapezius. The facial nerve innervates the muscles of facial expression; it is not found in the posterior triangle. The hypoglossal nerve provides motor innervation to the tongue--it is also not in the posterior triangle. Finally, the suprascapular nerve innervates supraspinatus and infraspinatus. It is located in the posterior triangle, and it might get injured in posterior neck operations, but the symptoms here are consistent with an injury to the accessory nerve.

An elderly patient developed fever and worsening headache a few days after sustaining a scalp laceration and subsequent infection due to a car accident. At the hospital the case was diagnosed as meningitis and superior sagittal sinus thrombosis. The attending physician suggested that infection to the sinus initially spread through one of the scalp layers. The scalp layer involved is: Areolar tissue Connective tissue Epicranial aponeurosis Periosteum Skin

Areolar tissue Areolar tissue is another name for the loose connective tissue layer of the scalp. Pus or blood can spread easily in this layer, and infections in this layer can pass into the cranial cavity through emissary veins. So, infections in the loose connective tissue can pass into intracranial structures such as the superior sagittal sinus, causing conditions like the superior sinus thrombosis. Remember--the scalp is comprised of the following layers, from superficial to deep: Skin, Connective tissue, Aponeurosis, Loose connective tissue, and Pericranium. (SCALP!) Although layer 2 is a connective tissue layer, too, this layer is thicker and it's not where infections can easily spread.

The entry of bacteria through which space could lead to an infection in the mastoid air cells: Auditory (nasopharyngeal) tube Cochlea External acoustic meatus Internal acoustic meatus Sacculus

Auditory (nasopharyngeal) tube The auditory (nasopharyngeal) tube is a connection between the nasal portion of the pharynx and the tympanic cavity that allows pressure to equalize on either side of the tympanic membrane. It is located in the anterior wall of the middle ear and is comprised of bone at the tympanic end and cartilage on the pharyngeal end. The pharyngeal mucosa is continuous with the lining of the tympanic cavity and mastoid air cells. This allows infectious material to pass to the middle ear and mastoid area. The cochlea is the organ of hearing that receives, interprets, and transmits sound via the vestibulocochlear nerve (CN VIII). The external acoustic meatus is the opening in the temporal bone that allows sound waves to reach the tympanic membrane. The internal acoustic meatus is the foramen in the temporal bone that allows the vestibulocochlear nerve and the facial nerve to pass into the skull at the base of the brain. Finally, the sacculus is a fluid filled sac that is part of the balancing apparatus of the ear - it is located in the vestibule of the ear.

Loss of sensation from the temporal region and loss of secretory function of the parotid gland would be caused by interruption of which nerve? Auriculotemporal Chorda tympani Deep temporal, posterior Facial Great auricular

Auriculotemporal nerve The auriculotemporal nerve is a branch of the mandibular division of the trigeminal nerve (V3). It has two important functions: First, it carries postganglionic parasympathetic fibers to the parotid gland. These fibers come from the otic ganglia, where they synapsed with the presynaptic fibers from the glossopharyngeal nerve (CN IX). Second, the auriculotemporal nerve provides sensory innervation to the skin of anterosuperior ear, part of the external auditory meatus, and the temporomandibular joint. So, the listed symptoms match with an injury to the auriculotemporal nerve. Chorda tympani is a branch of the facial nerve that provides secretomotor innervation to the submandibular and sublingual glands. It carries preganglionic parasympathetic axons to the submandibular ganglion. In the infratemporal fossa, chorda tympani joins the lingual nerve--it continues with the lingual nerve to the tongue where it supplies taste to the anterior 2/3 of the tongue. The posterior deep temporal nerve is a branch of the mandibular division of the trigeminal nerve which supplies motor innervation to temporalis. The facial nerve (CN VII) innervates all of the muscles of facial expression, and, through the chorda tympani, provides secremotor innervation to the submandibular and sublingual glands as well as taste sensation to the anterior 2/3 of the tongue. Finally, the great auricular nerve comes from the cervical plexus--it provides sensory innervation to the skin of the ear and the skin below the ear.

In repairing a damaged right subclavian artery, the surgeon notices and protects a large nerve encircling and passing around to the posterior surface of the artery. This nerve, which does not encircle the subclavian on the left side, is the: A. Phrenic nerve. B. Recurrent laryngeal nerve. C. Vagus nerve. D. Accessory nerve. E. Ansa cervicalis.

B. Recurrent laryngeal nerve.

In the Circle of Willis, the anterior communicating artery bridges: A. The basilar artery to the posterior cerebral artery. B. The left anterior cerebral artery to the right anterior cerebral artery. C. The anterior cerebral artery to the middle cerebral artery. D. The basilar artery to the middle cerebral artery. E. The internal carotid artery to the anterior cerebral artery.

B. The left anterior cerebral artery to the right anterior cerebral artery.

Which statements rearding the structures of the larynx and their relative position is correct in a runner who is breathing heavily during a race? A. Vocal folds are abducted and the rima glottidis is fully open. B. Vocal folds are adducted and the rima glottidis is closed. C. Vestibular folds are adducted and the rima vestibuli is closed. D. The epiglottis swings down to close the laryngeal inlet. E. None of the these are correct.

B. Vocal folds are adducted and the rima glottidis is closed.

The soft palate is active in all of the following except: Breathing Chewing Coughing Swallowing Yawning

Breathing The soft palate is the movable posterior 1/3 of the palate, which is suspended from the posterior border of the hard palate. When a person swallows, the soft palate is initially tensed to allow the tongue to press against it, squeezing the bolus of food to the back of the mouth. The soft palate is elevated posteriorly and superiorly against the wall of the pharynx, thereby preventing the passage of food into the nasal cavity. The soft palate functions similarly to prevent the bolus of food from passing into the nasal cavity while chewing and to prevent expectorated material from entering the nasal cavity while coughing. The soft palate also elevates when yawning. The soft palate does not elevate during breathing--if it did rise during breathing, the air inspired through the nose might be blocked from entering the trachea.

To drain an abscess (a closed collection of pus) affecting the cheek area, an emergency room physician used local anesthesia for the surgery. Which of the following nerves must be anesthetized because it carries pain sensation from the cheek area? buccal (V3) buccal (VII) inferior alveolar lingual mental

Buccal (V3) The buccal nerve is a branch of the mandibular division of the trigeminal nerve that transmits sensory information from the skin of the cheek area and the oral mucosa of the cheek. The buccal nerve is NOT the same as the buccal branches of the facial nerve, which are motor nerves that innervate the buccinator and the facial muscles of the upper lip. The buccal branches of the facial nerve do not have a sensory component! The inferior alveolar nerve is another branch of V3 that penetrates the mandibular foramen and is the sensory nerve for the mandible and all mandibular teeth. This is the nerve that dentists must anesthesize when they are working on the mandibular teeth. The lingual nerve is also a branch of V3--it transmits general sensation from the tongue. The lingual nerve also receives the chorda tympani, a branch of the facial nerve, which provides the lingual nerve with preganglionic parasympathetic fibers for the submandibular and and sublingual glands, as well as the taste fibers for the anterior 2/3 of the tongue. Finally, the mental nerve is a branch of V3 from the inferior alveolar nerve--it provides sensory innervation to the skin of the chin and lower lip.

While recovering from a right facial paralysis, a 36-year-old female patient complained that food accumulated between the teeth and the cheek mucosa when chewing. The deficiency of which muscle was most likely the cause of the chewing problem? Zygomaticus major Orbicularis oris Buccinator Levator labii superioris

Buccinator Although buccinator is innervated by the buccal branches of the facial nerve and not a branch of V3, the buccinator is an important muscle for mastication. The buccinator keeps the cheek taut so it is not folding over and becoming injured by chewing. It aids mastication by pulling the cheek against the molar teeth so that food does not keep collecting in the vestibule of the mouth. So, if this muscle was injured, the cheek could not press against the molar teeth, and food would fall between the teeth and cheek mucosa while chewing. The other 3 muscles are all innervated by the facial nerve, but they are important for facial expression not mastication. Zygomaticus major elevates and draws the corners of the mouth laterally. Orbicularis oris purses the lips. Levator labii superioris elevates the upper lip.

Which muscle will not be affected when the mandibular division of the trigeminal nerve (V3) is anesthetized? Anterior belly of digastric Buccinator Medial pterygoid Mylohyoid Temporalis

Buccinator Buccinator is innervated by the facial nerve (CN VII). It allows the corner of the mouth to be pulled laterally, and it allows the cheek to be pulled against the teeth. It is an important muscle for mastication, but it's not innervated by a branch of the mandibular division of the trigeminal nerve. (Note: the buccal branches of the facial nerve, which innervate buccinator, are not the same as the buccal nerve, a sensory branch of V3!) The other muscles listed are all innervated by branches of V3. Mylohyoid and the anterior belly of the digastric are innervated by a small branch of V3. They both elevate the hyoid and depress the mandible. Medial pterygoid and temporalis are muscles of mastication, and the muscles of mastication are innervated by V3.

A hydrocephalus (buildup of fluid in the ventricles) is diagnosed as a result of a blockage between the third and fourth ventricle. This is the: A. Lateral aperture. B. Interventricular foramen. C. Cerebral aqueduct. D. Arachnoid granulations. E. Median aperture.

C. Cerebral aqueduct.

The communication between the pharynx and the nasal cavity is known as the: Aditus Auditory tube Choanae Fauces Piriform recess

C. Choanae The choanae are the opening at the posterior border of the nasal cavity that allow the nasal cavity to communicate with the nasopharynx. The aditus is the laryngeal inlet - this is the space that is covered by the epiglottis when swallowing. The auditory tube is a tube that connects the nasopharynx with the middle ear, allowing for pressure to equalize on both sides of the tympanic membrane. The fauces is the passage from the mouth to the oropharynx, including the lumen and its boundaries. Finally, the piriform recess is a shallow depression located lateral to the aryepiglottic fold in the laryngopharynx. This is a place where food is commonly lodged.

The brain structure that is involved in auditory reflex is the: A. Superior colliculus. B. Corpus callosum. C. Inferior colliculus. D. Mammillary body. E. Pineal gland.

C. Inferior colliculus.

Which of the following is not innervated by the facial nerve? A. Zygomaticus major. B. Orbicularis oculi. C. Masseter. D. Buccinator. E. Depressor labii inferioris.

C. Masseter.

The superior belly of the omohyoid forms the anterior border of which cervical triangle? Carotid Muscular Omoclavicular Posterior Submandibular

Carotid The carotid triangle is bounded anteriorly by the superior belly of the omohyoid muscle, laterally by the sternocleidomastoid, and superiorly by the posterior belly of the digastric. The carotid vessels and carotid sheath can be found in this triangle. The muscular triangle is bounded by the midline as its medial border, the superior belly of the omohyoid as its superolateral border, and sternocleidomastoid as its inferolateral border. The strap muscles, thyroid, and parathyroids are found in this triangle. The omoclavicular triangle, in the posterior neck, is bounded superiorly by the inferior belly of the omohyoid muscle, anteriorly by the sternocleidomastoid muscle, and inferiorly by the clavicle. The third part of the subclavian artery and the subclavian vein are located in this triangle. The posterior triangle of the neck is bounded anteriorly by the sternocleidomastoid muscle, posteriorly by trapezius, and inferiorly by the clavicle. The submandibular triangle is bounded anteriorly by the anterior belly of the digastric, posteriorly by the posterior belly of the digastric, and superiorly by the lower border of the mandible. It contains the superficial portion of the submandibular gland and the facial artery and vein.

Following endarterectomy on the right common carotid, a patient is found to be blind in the right eye. It appears that a small thrombus embolized during surgery and lodged in the artery supplying the optic nerve. What artery would be blocked? Central artery of the retina Infraorbital Lacrimal Nasociliary Supraorbital

Central artery of the retina The central artery of the retina is a branch of the ophthalmic artery. It is the sole blood supply to the retina; it has no significant collateral circulation and blockage of this vessel leads to blindness. The branches of this artery are what you view during a fundoscopic exam. The infraorbital artery is a branch of the maxillary artery. It comes through the infraorbital foramen, inferior to the eye. It supplies the maxillary sinus, the maxillary incisors, canine and premolar teeth, and the skin of the cheek below the orbit. The supraorbital artery is another branch of the ophthalmic artery. It comes through the supraorbital foramen or notch and supplies blood to the muscles, skin and fascia of the forehead. The lacrimal artery is a branch of the ophthalmic artery that supplies the lacrimal gland. The nasociliary artery doesn't exist, but there is a nasociliary nerve (the third and lowest branch of the ophthalmic division) that travels with the continuation of the ophthalmic artery.

An infant was found to have hydrocephalus. Studies revealed that the hydrocephalus was caused because CSF could not get out of the third ventricle. The passage blocked was the: Central canal Cerebral aqueduct Interventricular foramen Lateral foramen (of Luschka) Medial foramen (of Magendie)

Cerebral aqueduct For CSF to travel from the third ventricle to the 4th ventricle and the central canal of the spinal cord, it must pass through the cerebral aqueduct. So, this is the passageway that must be blocked. The central canal is the space where CSF flows through the spinal cord. It is continuous with the 4th ventricle. The foramen of Luschka and foramen of Magendie are small foramina in the 4th ventricle that allow the CSF to leave the ventricular system and enter the subarachnoid space. The interventricular foramina are passages in the lateral ventricles that allow the CSF to leave the lateral ventricles and enter the 3rd ventricles.

The laryngeal muscle most responsible for stretching (elongating) the vocal ligament is the : Posterior cricoarytenoid Lateral cricoarytenoid Thyroarytenoid Arytenoid Cricothyroid

Cricothyroid Cricothyroid draws the thyroid cartilage forward, tensing the vocal ligaments. This is the one muscle innervated by the external branch of the superior laryngeal nerve. Posterior cricoarytenoid is a very important muscle; it's the only muscle that abducts the vocal folds. This muscle is innervated by the inferior laryngeal nerve, which is a continuation of the recurrent laryngeal nerve. If this muscle is denervated, the vocal folds may be paralyzed in an adducted position, which would prevent air from entering the trachea. Lateral cricoarytenoid, thyroarytenoid, and arytenoid are all muscles that adduct the vocal folds. They are all innervated by the inferior laryngeal nerve.

A deep laceration of the face in the middle of the parotid gland could affect the: A. Lingual artery. B. Facial artery. C. Hypoglossal nerve. D. Facial nerve. E. External jugular vein.

D. Facial nerve.

The only cranial nerve to originate from the dorsal midbrain is the __________. A. Oculomotor nerve (III). B. Trigeminal nerve (V). C. Facial nerve (VII). D. Trochlear nerve (IV). E. Abducens nerve (VI).

D. Trochlear nerve (IV).

Which of the following cranial nerves contain ONLY general efferent (somatic motor) fibers? A. Optic nerve. B. Trigeminal nerve. C. Facial nerve. D. Trochlear nerve. E. Glossopharyngeal nerve.

D. Trochlear nerve.

In the above picture, which muscle is acting to raise the corner of the mouth laterally and superiorly? A. Levator anguli oris. B. Levator labii superioris C. Orbicularis oris. D. Zygomaticus major. E. Nasalis.

D. Zygomaticus major.

Description for the following questions: Examination of a patient with an ulcerative carcinoma of the posterior third of the tongue revealed bleeding from the lesion and difficulty swallowing (dysphagia). The bleeding was seen to be arterial, which of the following arteries was involved? Deep lingual Dorsal lingual Facial Sublingual Tonsillar

Dorsal lingual artery The dorsal lingual artery runs on the superficial surface of the tongue--it is a branch of the lingual artery that delivers blood to the posterior superficial tongue. So, this artery must be the source of the hemorrhage. The deep lingual artery and sublingual artery are two terminal branches of the lingual artery. These branches run in the floor of the mouth (sublingual) and the deep surface of the tongue (deep lingual).

A patient has sustained a fracture to the base of the skull. Thorough examination concluded that the right greater petrosal nerve, among other structures, has been injured. This conclusion was based on which of the patient's signs: Partial dryness of the mouth due to lack of salivary secretions from the submandibular and sublingual glands Partial dryness of the mouth due to lack of salivary secretions from the parotid gland Dryness of the right cornea due to lack of lacrimal secretion Loss of taste sensation from the right anterior 2/3rd of the tongue Loss of general sensation from the right anterior 2/3rd of the tongue

Dryness of the right cornea due to lack of lacrimal secretion. The greater petrosal nerve is a branch of the facial nerve that arises adjacent to the geniculate ganglion. It carries preganglionic parasympathetic fibers to the pterygopalatine ganglion. The fibers synapse in this ganglion, and the postsynaptic parasympathetic fibers go on to innervate the lacrimal gland, mucous glands of the palate, and the mucous glands of the nasal cavity. So, if the greater petrosal nerve was damaged, the presynaptic parasympathetic fibers would be lost, and the lacrimal gland would not function properly. The sublingual and submandibular glands are innervated by the chorda tympani, which brings preganglionic parasympathetic fibers to the submandibular ganglion. The chorda tympani also provides taste sensation to the anterior 2/3 of the tongue. Although the chorda tympani, like the greater petrosal nerve, is a branch of the facial nerve, the chorda tympani never runs with the greater petrosal nerve. So, injuring the greater petrosal nerve would not harm the chorda tympani. The parotid gland is innervated by the lesser petrosal nerve, a branch of the glossopharyngeal nerve (CN IX). The fibers from this nerve go to the otic ganglion, synapse there, and then continue on to the parotid gland via the auriculotemporal nerve. Finally, general sensation to the anterior 2/3 of the tongue is carried by the lingual nerve, a branch of V3.

A nursing infant develops a powerful sucking muscle that adults also use for whistling. This muscle is the: A. Mentalis. B. Zygomaticus major. C. Orbicularis oris. D. Platysma. E. Buccinator.

E. Buccinator.

Broca's area, which controls the motor movements necessary for speaking, is located in the: A. Occipital lobe. B. Insula lobe. C. Temporal lobe. D. Parietal lobe. E. Frontal lobe.

E. Frontal lobe.

What is the area of the laryngeal cavity above the vestibular fold of the larynx? A. Laryngeal ventricle. B. Subglottal atrium. C. Saccule. D. Infraglottic cavity. E. Laryngeal vestibule.

E. Laryngeal vestibule.

The brain region where there is a crossover of efferent (motor) fibers is: A. Pons. B. Midbrain. C. Diencephalon. D. Cerebellum. E. Medulla oblongata.

E. Medulla oblongata.

The only cranial nerve that does not pass through the thalamus is: A. Glossopharyngeal nerve (IX). B. Optic nerve (II). C. Facial nerve (VII). D. Vestibulocochlear nerve (VIII). E. Olfactory nerve (I).

E. Olfactory nerve (I).

Which of the following cerebral regions is found in the parietal lobe? A. The primary visual cortex. B. The insula cortex. C. Broca's area. D. The primary motor cortex. E. The primary somatosensory cortex.

E. The primary somatosensory cortex.

Which nerve provides motor innervation to the masseter muscle? A. Glossopharyngeal nerve. B. Facial nerve. C. Vagus nerve. D. Hypoglossal nerve. E. Trigeminal nerve.

E. Trigeminal nerve.

Which of the following cranial nerves strictly have motor function? A. Glossopharyngeal nerve (IX). B. Trigeminal nerve (V). C. Optic nerve (II). D. Olfactory nerve (I). E. Trochlear nerve (IV).

E. Trochlear nerve (IV).

Following surgery on the upper pole of the right lobe of the thyroid gland, a patient complains of hoarseness and weakness of voice. What nerve may have been injured? Ansa cervicalis External branch of the superior laryngeal Inferior laryngeal Internal branch of the superior laryngeal Recurrent laryngeal

Exernal branch of the superior laryngeal The external branch of the superior laryngeal nerve travels with the superior thyroid artery toward the superior pole of the thyroid. So, it's in the right place to be injured by surgery on the upper pole of the gland. The external branch of the superior laryngeal nerve innervates cricothyroid, the muscle responsible for elongating the vocal cords. The patient's symptoms of horseness and a weak voice would fit with an injury to this nerve and the denervation of this muscle.

During a vaginal delivery, a surgeon performed median episiotomy in which he cut too far (i.e., through the perineal body into the structure immediately posterior). Which perineal structure did he cut? Bulbospongiosis muscle External anal sphincter muscle Ischiocavernosis muscle Sacrospinous ligament Sphincter urethrae

External anal sphincter muscle An episiotomy is an incision made in the perineum to enlarge the distal end of the birth canal and to prevent serious damage to the perineal structures. This procedure is often performed when there is a risk of tearing the birth canal due to a breech or forceps delivery. When performing a median episiotomy, a cut is made immediately posterior to the vagina, through the perineal body. If this cut went too far, the physician might cut through the external anal sphincter or the rectum. So, external anal sphincter is the correct answer.

During an intramural baseball game a player is hit in the side of the head, between the eye and the ear. He immediately loses consciousness, wakes up momentarily and then becomes comatose. He is rushed to the ER and is immediately given a CT scan. The scan shows a skull fracture and an accumulation of blood between the dura and the cranial bone on the side of his head, compressing his cerebrum. He is rushed to surgery where a hole is bored into his skull to relieve the pressure. After a few tense hours, he regains consciousness and has an uneventful recovery. The hemorrhage from the fracture would be described as: Epidural Intracerebral Subaponeurotic Subarachnoid Subdural

Epidural An epidural hemorrhage is a hemorrhage of blood into the space between the dura and the skull. These hemorrhages are usually caused by rupturing the middle meningeal artery, which supplies blood to the dura and the bones of the cranial vault. This hemorrhage results in compression of the dura mater and the brain; if it is not drained, it may result in the brain herniating through the tentorium and death. An intracerebral hemorrhage is a hemorrhage within the cerebral hemispheres. A subaponeurotic hemorrhage could be a collection of blood under the aponeurosis of the scalp, but this is not really a brain hemorrhage and is not as clinically significant as the other answer choices. A subarachnoid hemorrhage is an acute condition where blood collects in the area between the pia mater and arachnoid mater. This is often secondary to a head injury or a ruptured aneurysm. A subdural hemorrhage is characterised by a collection of blood beneath the dura, often caused by a head injury.

Any manipulation of the superior thyroid artery must be undertaken with care not to damage its small companion nerve, the: Cervical sympathetic trunk External branch of the superior laryngeal Inferior root of the ansa cervicalis Internal branch of the superior laryngeal Recurrent laryngeal

External branch of the superior laryngeal The external branch of the superior laryngeal nerve runs with the superior thyroid artery--this artery and nerve might be damaged when removing the superior pole of the thyroid. If this nerve was damaged, the cricothyroid muscle would be paralyzed, and a patient would be unable to tense the vocal cords. The internal branch of the superior laryngeal nerve runs with the superior laryngeal artery and provides sensory innervation to the mucous membrane of the larynx, superior to the vocal fold. The recurrent laryngeal nerve crosses the inferior thyroid artery, near the lower pole of the thyroid. This means that the recurrent laryngeal nerve would be at risk in any surgery involving the inferior thyroid artery or the inferior poles of the thyroid. The recurrent laryngeal innervates all the muscles of the larynx with the exception of cricothyroid. So, an injury to the recurrent laryngeal nerve might lead to hoarseness and dificulty breathing. The cervical sympathetic trunk lies in front of the prevertebral muscles in the prevertebral fascia--it would not be near the structures involved with thyroid surgery. The inferior root of ansa cervicalis is a branch of the cervical plexus. It innervates the strap muscles. Ansa cervicalis hangs in a loop over the carotid vessels; it is not associated with the thyroid gland.

An emergency room patient presented with considerable bleeding from a shallow stab wound in the omoclavicular triangle. Which vessel was most likely cut? Axillary vein Common carotid artery External jugular vein Inferior thyroid vein Superior thyroid artery

External jugular vein The external jugular vein is a superficial vein on the lateral side of the neck. It is in the superficial fascia, so it could be easily injured by a superficial wound. Since this vein travels in the omoclavicular triangle, this is the structure that was injured. The axillary vein is the continuation of the subclavian vein lateral to the first rib--it is important in draining the upper limb, but it's not associated with the triangles of the neck. The common carotid artery is found in the carotid sheath and carotid triangle. The inferior thyroid vein and superior thyroid artery are both found near the thyroid gland; therefore, they are both in the muscular triangle.

You have been asked to assess the neurological deficit that might exist in a patient diagnosed with cavernous sinus thrombosis. You will focus your examination on cranial nerves related to the sinus that includes all the following EXCEPT: Abducens (CN VI) Facial (CN VII) Oculomotor (CN III) Ophthalmic division of the trigeminal nerve (CN V1) Trochlear (CN IV)

Facial (CN VII) The cavernous sinus is a venous sinus of the brain, lateral to the body of the sphenoid bone. All of the cranial nerves and vessels that pass out of the skull at the superior orbital fissure pass through the cavernous sinus. This includes the oculomotor nerve, the trochlear nerve, the ophthalmic division of the trigeminal nerve, and the abducens nerve. The internal carotid artery also passes through the cavernous sinus. Since three of the nerves in the cavernous sinus control the motions of the extraocular muscles, testing eye movements would be a good way to see if nerves in the cavernous sinus were disrupted. The ophthamic division of the trigeminal nerve, which supplies cutaneous sensation to the skin of the upper face, could be tested by evaluating the sensations on the forehead. The facial nerve is not associated with the cavernous sinus. It passes through the internal acoustic meatus and exits the skull through the stylomastoid foramen. So, you would not need to test to see if the facial nerve was intact.

The geniculate ganglion is the sensory ganglion of which nerve: Facial Glossopharyngeal Trigeminal Vagus Vestibulocochlear

Facial nerve The geniculate ganglion is the sensory ganglion of the facial nerve. These fibers travel as part of the chorda tympani to provide taste sensation to the anterior 2/3 of the tongue. The glossopharyngeal nerve has superior and inferior ganglia, located near the jugular foramen. These ganglia contain the cell bodies for the afferent components of this nerve. The trigeminal nerve has a trigeminal ganglion which contains the cell bodies of neurons composing the sensory root of this nerve. The vagus nerve also has a superior and inferior ganglion, which are located near the ganglia of the glossopharyngeal nerve. The superior ganglion is concerned with the general sensory component of the nerve, and the inferior ganglion is concerned with the visceral sensory component of the nerve. Finally, the vestibulocochlear nerve has a vestibular ganglion, which houses the neurons for balance, and the cochlear ganglion, which houses the neurons concerned with hearing.

Blockage of the flow of cerebrospinal fluid (CSF) within the cerebral aqueduct (of Sylvius) normally would result in the enlargement of all of the following ventricular spaces except the: Fourth ventricle Interventricular foramen (of Monro) Lateral ventricle Third ventricle

Fourth ventricle If the cerebral aqueduct was blocked, CSF would not flow from the third ventricle into the fourth ventricle and the central canal. So, all the spaces proximal to the blockage would enlarge, while all the spaces distal to the blockage would be normal. The third ventricle, lateral ventricle, and interventricular foramen are proximal to the blockage of CSF; fluid circulates from the lateral ventricles, through the interventricular foramen, to the third ventricle before reaching the cerebral aqueduct. This means that all of these spaces should be enlarged with fluid. The fourth ventricle, however, needs the cerebral aqueduct to be open so that it can receive fluid--it would not become distended from the blockage in the aqueduct.

Damage of the lingual nerve before it is joined by the chorda tympani in the infratemporal fossa would cause loss of: general sensation to the anterior two thirds of the tongue general sensation to the posterior one third of the tongue secretion of the submandibular gland taste sensation from the anterior two thirds of the tongue taste sensation from the posterior one third of the tongue

General sensation to the anterior 2/3 of the tongue The lingual nerve is a branch of V3, the mandibular division of the trigeminal nerve. It transmits general sensation from the anterior 2/3 of the tongue. Damaging this nerve anywhere along its course would cause someone to lose general sensation to the anterior 2/3 of the tongue. The chorda tympani is a branch of VII--it carries taste fibers to the anterior 2/3 of the tongue and presynaptic parasympathetic fibers to the submandibular ganglion. The fibers from the chorda tympani join the lingual nerve as they travel to the submandibular ganglion and the anterior tongue. However, if the lingual nerve was damaged before the chorda tympani had joined it, the chorda tympani fibers would still be intact. So, there would still be taste innervation to the anterior 2/3 of the tongue and the submandibular gland would still secrete. If the lingual nerve was disrupted after the chorda tympani had already joined it, there would be no secretomotor innervation to the submandibular and sublingual glands, and there would be no taste or general sensation to the anterior 2/3 of the tongue. As for the posterior 1/3 of the tongue, it receives taste and general sensation from the glossopharyngeal nerve (CN IX).

The carotid body is innervated by a branch of the: Glossopharyngeal nerve Hypoglossal nerve Spinal accessory nerve Sympathetic trunk Vagus nerve

Glossopharyngeal nerve (CN IX) The carotid body is innervated by the carotid branch of the glossopharyngeal nerve. It is a small, reddish-brown ovoid mass that lies on the medial side of the carotid bifurcation, serving as a chemoreceptor that monitors the level of carbon dioxide in the blood. The hypoglossal nerve (CN XII) innervates the muscles of the tongue. The spinal accessory nerve is a motor nerve that innervates the sternocleidomastoid and the trapezius. The sympathetic trunk provides sympathetic innervation to the head and neck. Finally, the vagus nerve supplies motor innervation to the muscles of the pharynx and larynx, with the exception of stylopharyngeus, and the muscles of the palate, with the exception of tensor veli palatini.

A 20 year old man was brought into the emergency room with a stab wound in the upper part of the neck. Although there was no major damage done, he lost sensation from the skin over the angle of the jaw. Which nerve has been cut? Supraclavicular n. Transverse cervical n. Great auricular n. Greater occipital n. Lesser occipital n.

Great auricular n. The great auricular nerve comes from C2 and C3 branches of the cervical plexus; it provides sensory innervation to the ear and the skin below the ear, including the angle of the jaw. The supraclavicular nerves come from C3 and C4 in the cervical plexus. These cutaneous nerves provide sensory innervation to the skin of the root of the neck, upper chest, and upper shoulder. The transverse cervical nerves also come from the cervical plexus--they are from C2 and C3 and provide sensory innervation to the skin of the neck anteriorly. Finally, the lesser occipital nerve comes from C2--it innervates the skin behind the ear. The greater occipital nerve is not from the cervical plexus, which is the ventral rami of C1-4--it is the cutaneous branch of the DORSAL primary ramus of spinal nerve C2. It provides cutaneous innervation to the posterior scalp.

While doing a postoperative physical on a patient who has undergone carotid endarterectomy on the right side, it was noted that the tongue deviated toward the right when the patient was asked to point the tongue outward. What nerve crossing the carotid artery must have been injured? Glossopharyngeal Hypoglossal Inferior alveolar Lingual Vagus

Hypoglossal The hypoglossal nerve crosses the carotid artery before diving into the floor of the mouth, so it might get injured during this procedure. This nerve innervates the muscles of the tongue. When the hypoglossal nerve is injured and a patient sticks out the tongue, the tongue points to the side with the injured nerve. So, this patient, who had the right side endarctectomy, has an injury to the right hypoglossal nerve. The symptoms that you see in this case should point you toward suspecting damage to the hypoglossal nerve. The glossopharyngeal nerve crosses around the internal carotid artery and the stylopharyngeus muscle--it could theoretically be injured in this procedure, but the symptoms are not consistent with an injury to this structure. The inferior alveolar nerve and lingual nerves are branches of the mandibular division of the trigeminal nerve. They are not associated with the carotid vessels. The vagus nerve is found in the carotid sheath. This nerve can be easily injured by an endarterectomy, although damage to the vagus would cause problems with swallowing, not with tongue movements.

The pharyngeal tonsils, or adenoids, may become inflamed and in serious cases need to be removed. Where would the physician search for them? In the tonsillar fossa In the pharyngeal recess In the piriform recess In the roof of the nasopharynx Upon the dorsal surface of the tongue

In the roof of the nasopharynx The pharyngeal tonsil is located on the roof of the nasopharynx. The pharyngeal recess is a space located posterior to the torus tubarius in nasopharynx. The tonsillar fossa is the space where the palatine tonsil is located--it is found between the palatoglossal fold and the palatopharyngeal fold. The piriform recess is a shallow depression located lateral to the aryepiglottic fold in the laryngopharynx. This is a common place where food can get stuck. The piriform recess is also significant because the internal branch of the superior laryngeal nerve is located immediately deep to the mucosa of this region--so, an injury here may damage this nerve! Finally, the lingual tonsil is found on the dorsal surface of the tongue.

An elderly patient with chronic otitis media (middle ear infection) might have all the following complications EXCEPT: Inabilty to chew food due to injury to the mandibular division of the trigeminal nerve (CN V) Loss of taste in the anterior part of the tongue due to injury to the chorda tympani nerve Mastoiditis Paralysis of facial muscles due an injury to the facial nerve (CN VII) Some degree of deafness due to damage to the ossicles

Inability to chew food due to injury to the mandibular division of the trigeminal nerve (CN V3) The mandibular division of the trigeminal nerve is not associated with the middle ear.

A 38-year-old patient complained of acute dental pain. The attending dentist found penetrating dental caries (dental decay) affecting one of the mandibular molar teeth. Which nerve would the dentist need to anesthetize to work on that tooth? Lingual Inferior alveolar Buccal Mental Mylohyoid

Inferior alveolar The inferior alveolar nerve is a branch of the mandibular division of the trigeminal nerve (V3). It penetrates the mandibular foramen and is the sensory nerve for the mandible and all mandibular teeth. This is the nerve anesthetized by dentists working on the mandibular teeth. It is anesthetized near the mandibular foramen, so sensory nerves from branches distal to that point would be blocked. The lingual nerve is another branch of V3--it transmits general sensation from the tongue. The lingual nerve also receives the chorda tympani, a branch of the facial nerve. The chorda tympani gives the lingual nerve preganglionic parasympathetic fibers for the submandibular and sublingual glands, as well as the taste fibers for the anterior 2/3 of the tongue. The buccal nerve is a branch of V3 that transmits sensory information from the skin of the cheek area and the oral mucosa of the cheek. Remember: The buccal nerve is NOT the same as the buccal branch of the facial nerve, which is a motor nerve innervating the buccinator and muscles of the upper lip!!!

In explaining the pain caused by the drilling of a mandibular molar tooth crown to a freshman medical student, a dentist identified the nerve conducting the pain sensations as the: Lingual Mylohyoid Inferior alveolar Buccal

Inferior alveolar The inferior alveolar nerve is a branch of the mandibular division of the trigeminal nerve (V3). It travels through the mandibular foramen and provides sensory innervation to the mandibular teeth. The lingual nerve is another branch of V3--it travels in the floor of the mouth and provides sensory innervation to the anterior 2/3 of the tongue and the floor of the mouth. The nerve to mylohyoid is a motor branch of V3--it provides motor innervation to mylohyoid and the anterior belly of the digastric. Finally, the buccal nerve is a sensory branch of V3 that supplies the cheek and oral mucosa.

The nerve that innervates all but one muscle of larynx is the: Glossopharyngeal nerve, pharyngeal branch Inferior laryngeal nerve Superior laryngeal nerve, external branch Superior laryngeal nerve, internal branch Thyrohyoid nerve

Inferior laryngeal nerve The inferior laryngeal nerve is the continuation of the recurrent laryngeal nerve, superior to the inferior border of the cricoid cartilage. It innervates the posterior cricoarytenoid muscle, which is responsible for abducting the vocal folds. The inferior laryngeal nerve also innervates all of the other intrinsic muscles of the larynx, with the exception of the cricothyroid muscle. The pharyngeal branch of the glossopharyngeal nerve provides sensory innervation to the pharynx, but not to the larynx. The glossopharyngeal nerve only provides motor innervation to stylopharyngeus. The external branch of the superior laryngeal nerve innervates the cricothyroid muscle, which is responsible for stretching the vocal ligaments. The internal branch of the superior laryngeal nerve is responsible for providing sensory innervation to the mucosa of the larynx, superior to the true vocal fold. The thyrohyoid nerve is a branch of ansa cervicalis-- it innervates the thyrohyoid muscle, which elevates the larynx and depresses the hyoid bone (or helps to fix the hyoid in space, so that other muscles, such as tongue muscles, can work).

A 23-year-old man comes to you complaining that he can't stop crying, i.e. tears regularly run down the right side of his face. You suspect that one of the lacrimal ducts on the right side of the face is blocked. You look into an endoscope to see if the nasolacrimal duct is blocked. Into which part of the nasal cavity would you look to see the opening of the duct? Hiatus semilunaris Inferior meatus Middle meatus Sphenoethmoidal recess Superior meatus

Inferior meatus The inferior meatus receives the nasolacrimal duct which conducts tears from the orbit. So, this is the place where you should be looking to see the opening of the nasolacrimal duct. The semilunar hiatus is found in the middle meatus--it receives drainage from the frontonasal duct (draining the frontal sinus), the anterior ethmoidal air cells, and the maxillary sinus. The sphenoethmoid recess is the opening for the sphenoethmoidal sinus. The superior meatus is the opening for the posterior ethmoidal air cells.

The extraocular muscle that does not originate at or near the apex of the orbit is the : Inferior oblique Inferior rectus Levator palpebrae superioris Superior oblique Superior rectus

Inferior oblique The inferior oblique muscle does not originate at the apex of the orbit. It takes origin from the floor of the orbit, lateral to the lacrimal groove. The inferior rectus and superior rectus muscles take origin from the common tendinous ring at the apex of the orbit. The levator palpebrae superioris takes origin from the apex of the orbit above the optic canal. The superior oblique muscle takes origin from the apex of the orbit, above the optic canal.

The middle cervical sympathetic ganglion, recurrent laryngeal nerve, and parathyroid glands are all closely related with which blood vessel? Inferior thyroid artery Inferior thyroid vein Superior thyroid artery Superior thyroid vein

Inferior thyroid artery The inferior thyroid artery branches from the thyrocervical trunk to provide blood to the lower poles of the thyroid gland. This artery approaches the posterior side of the thyroid gland, crossing the recurrent laryngeal nerve and coming near the middle cervical sympathetic ganglion. This artery also supplies blood to the parathyroid glands, which are on the posterior surface of the thyroid. The inferior thyroid veins are associated with the lower parts of the gland--however, these veins are on the anterior side of the thyroid and are not as closely associated with these structures found deep to the gland. The superior thyroid artery and vein are associated with the superior pole of the thyroid. Remember--the important relationship here is that the superior thyroid artery travels with the external branch of the superior laryngeal nerve.

The location of the otic ganglion is in the: Pterygopalatine fossa Internal ear Infratemporal fossa Middle cranial fossa None of the above

Infratemporal fossa

Following thyroid surgery, it was noted that a patient frequently aspirated fluid into her lungs. Upon examination it was determined that the area of the piriform recess above the vocal fold of the larynx was numb. What nerve may have been injured? External branch of the superior laryngeal Hypoglossal Internal branch of the superior laryngeal Lingual Recurrent laryngeal

Internal branch of the superior laryngeal The internal branch of the superior laryngeal is a sensory nerve that pierces the thyrohyoid membrane along with the superior laryngeal artery. It supplies sensory fibers to the mucous membrane of the larynx, superior to the vocal folds. Since this area lost sensation, it appears that the internal branch of the superior laryngeal nerve must have been injured. The external branch of the superior laryngeal nerve is a motor nerve that innervates the cricothyroid muscle--it does not provide any sensory innervation to the larynx. The recurrent laryngeal nerve ascends from the thorax and provides motor innervation to the upper esophagus, lower pharynx, and all the laryngeal muscles except cricothyroideus. The hypoglossal nerve supplies motor innervation to the muscles of the tongue. The lingual nerve is a sensory nerve for the anterior 2/3 of the tongue. These nerves are not important for innervating the larynx.

If the venous drainage of the anal canal above the pectinate line is impaired in a patient with portal hypertension, there may be an increase in blood flow downward to the systemic venous system via anastomoses with the inferior rectal vein, which is a tributary of the: External iliac Inferior gluteal Inferior mesenteric Internal iliac Internal pudendal

Internal pudendal The rectal venous plexus is one of the four portal/systemic anastomoses. Blood from the portal system can flow into the venous system at this junction. This means that portal blood, from the superior rectal vein, could flow through the rectal venous plexus, into the inferior rectal vein and into the systemic venous drainage. Now, you just need to figure out what the inferior rectal vein drains into. And it drains into the internal pudendal vein, so that's the answer.

In order to perform an episiotomy prior to childbirth, the perineum must be anesthetized. By inserting a finger in the vagina and pressing laterally, what palpable bony landmark can be used as the posterior limit of the pudendal canal? Coccyx Ischial tuberosity Ischiopubic ramus Obturator groove Ischial spine

Ischial spine The pudendal canal travels from the lesser sciatic foramen to the deep transverse perineus muscle. The ischial spine marks the posterior limit of the pudendal canal, so that's the correct answer. If you weren't sure about that, you might also notice that the physician here is trying to perform a transvaginal pudendal nerve block. This means that the physician will be using the ischial spine as a landmark and inserting the needle near this prominence, coating the pudendal nerve with anesthesia before it gives off its branches.

During childbirth a bilateral pudendal nerve block may be performed to provide anesthesia to the majority of the perineum and the lower one fourth of the vagina. To do this an anesthetic agent is injected near the pudendal nerve as it passes from the pelvic cavity to the perineum. The physician inserts a finger into the vagina and presses laterally to palpate what landmark? Arcus tendineus levator ani Coccyx Ischial spine Lateral fornix Obturator foramen

Ischial spine When performing a transvaginal pudendal nerve block, the ischial spine is palpated through the wall of the vagina and the needle is then passed through the vaginal mucous membrane toward the ischial spine. Eventually, the needle pierces the sacrospinous ligament, at which point the pudendal nerve is bathed with anesthetic. Remember--the pudendal nerve is within the pudendal canal, and it wraps around the ischial spine before it delivers its branches. So, administering the nerve block at the ischial spine allows a physician to anesthetize all the branches of the pudendal nerve. This is a very important landmark that you want to remember!

Which of the following statements about the cervical plexus is true? It comprises spinal segments C1-C3 It consists solely of cutaneous nerves It consists solely of ventral rami of spinal nerves It supplies motor innervation to platysma

It consists solely of ventral rami of spinal nerves The cervical plexus consists solely of the ventral primary rami of spinal nerves C1-C4. It supplies cutaneous innervation through nerves like the lesser occipital, great auricular, supraclavicular and transverse cervical. It also supplies motor innervation to the strap muscles through the ansa cervicalis. So, it has a motor and sensory component. It does not innervate the platysma--platysma is a muscle of facial expression innervated by the cervical branch of the facial nerve (CN VII).

Which statement is true of the internal jugular vein? It drains all of the thyroid gland on that side of the body It drains into the external jugular vein It is accompanied by the deep cervical chain of lymph nodes It lies deep to the prevertebral fascia It passes superficial to the sternocleidomastoid muscle

It is accompanied by the deep cervical chain of lymph nodes The deep cervical chain of lymph nodes runs closely with the internal jugular vein. The internal jugular vein does not drain all of the thyroid gland--the superior and middle thyroid veins drain into the internal jugular vein, but the inferior thyroid veins drain directly into the brachiocephalic veins. The internal jugular vein does not drain into the external jugular vein. The external jugular vein is a smaller vein on the lateral neck which drains into the subclavian vein. The internal jugular vein meets the subclavian vein, and the two vessels join to form the brachiocephalic vein. The prevertebral fascia is found deep in the neck, over the vertebrae and the prevertebral muscles. So, the internal jugular vein is very superficial to this fascia. Finally, the sternocleidomastoid muscle crosses over the internal jugular vein.

The glossopharyngeal nerve exits the skull via what opening? Foramen ovale Carotid canal Jugular foramen Hypoglossal canal Stylomastoid foramen

Jugular foramen The glossopharyngeal nerve (CN IX), vagus (CN X) and accessory nerve (CN XI) exit the skull at the jugular foramen. The posterior meningeal artery enters the skull through this space. The mandibular division of the trigeminal nerve (V3) exits the skull through foramen ovale. The carotid canal is the place where the internal carotid artery and the internal carotid nerve plexus enter the skull. The hypoglossal canal is where the hypoglossal nerve (CN XII) leaves the skull. The stylomastoid foramen is the hole that the facial nerve (CN VII) uses to exit the skull.

A patient is brought into the Emergency Room in respiratory distress. It is quickly decided to create an emergency airway to restore respiration. At what level could you rapidly create an airway below the vocal cords with a minimum danger of hemorrhage? Just above the jugular notch Just above the thyroid cartilage Just below the cricoid cartilage Just below the thyroid cartilage Through the 3rd tracheal ring

Just below the thyroid cartilage To create an emergency airway, a midline cut can be made through the cricothyroid ligament and membrane. So, an opening is created between the thyroid cartilage and the cricoid cartilage. Since this is below the glottis and the tracheal inlet (which is the usual place where things get lodged), this is a good spot to make the cut. It is also easy to find the thyroid cartilage on a patient and make the incision right below that spot.

Incapacity to protrude the mandible indicates a dysfunction of which muscle? Anterior belly of digastric Buccinator Lateral pterygoid Mylohyoid Temporalis

Lateral pterygoid The lateral pterygoid muscle protrudes the mandible--it pulls the mandible forward to allow for depression of the chin (which is mostly produced by gravity). None of the other muscles help with this function. The anterior belly of the digastric and mylohyoid have similar functions: they both help elevate the hyoid bone and depress the mandible. The buccinator is a muscle in the cheek; it pulls the corner of mouth laterally and presses the cheek against the teeth. Temporalis is important for retracting and elevating the mandible.

In dislocation of the jaw, displacement of the articular disc beyond the articular tubercle of the temporomandibular joint results from excessive contraction of which muscle? Buccinator Lateral pterygoid Medial pterygoid Masseter Temporalis

Lateral pterygoids The TMJ may dislocate anteriorly due to excessive contraction of the lateral pterygoids (for example, during excessive opening of the mouth). This could happen if someone yawned too much or took a large bite of food. Posterior dislocations of the TMJ are rare due to resistance from the postglenoid tubercle and the strong lateral ligaments. The buccinator is an important muscle for mastication because it keeps the cheek taut, so the cheek can press against the molars. However, it is a facial muscle innervated by the facial nerve, and it is not active at the TMJ. The medial pterygoid, masseter and temporalis produce motion at the TMJ, but contracting these muscles does not cause the joint to dislocate.

A 46-year-old female patient comes to the emergency department complaining of pain in the area just below her mandible on the right side of her face. She says that the pain is particularly severe when she eats. The area of the submandibular gland is tender and swollen, as is the area in the floor of her mouth lateral to the tongue. You suspect a stone in the submandibular duct, and a plain film radiograph shows a density in that region consistent with a stone. In order to remove the stone, the duct must be incised in the floor of the mouth. What nerve, that loops around the duct, is in danger in such an incision? Chorda Tympani Glossopharyngeal Hypoglossal Internal branch of the superior laryngeal Lingual

Lingual The lingual nerve is found in the floor of the mouth--it wraps around the submandibular duct. So that nerve might be injured as you try to remove the stone from the submandibular duct. The lingual nerve contains general sensory fibers for the anterior 2/3 of the tongue, and fibers from the chorda tympani that provide taste to the anterior 2/3 of the tongue. All of these sensory components might be lost if the lingual nerve was damaged. At the point where the lingual nerve is wrapping around the submandibular gland, the fibers from chorda tympani have already joined the lingual nerve.This means that fibers from chorda tympani might be damaged, but the chorda tympani itself is not near the submandibular duct. The glossopharyngeal nerve provides sensory and taste innervation to the posterior 1/3 of the tongue and sensory innervation to the pharynx--it is not associated with the submandbular duct. The hypoglossal nerve is in the sublingual space, just like the submandibular duct. However, it is deeper in the sublingual space and does not wrap around the duct. Remember--the lingual nerve is a much more superficial structure in the sublingual space. Finally, the internal branch of the superior laryngeal nerve crosses the thyrohyoid membrane to provide sensory innervation to the mucosa of the pharynx, superior to the vocal folds.

Sympathetic fibers reach the tongue by way of the: lingual nerve maxillary artery hypoglossal nerve lingual artery glossopharyngeal nerve

Lingual artery Remember--all over the body, the vasculature is the number one target of sympathetic nerves. Sympathetic nerves help to constrict the vasculature, and they are found covering arteries in periarterial plexuses. In the head, sympathetics travel to targets on vessels. So, you know that the sympathetic nerves are coming to the tongue on an artery--now, you just need to determine what artery goes to the tongue. And that artery is the lingual artery. The lingual artery is the second branch off the anterior side of the external carotid artery. It travels in the floor of the mouth and supplies blood to the tongue, suprahyoid muscles, and the palatine tonsil. This is the artery going to the targeted area, so that's the answer you're looking for. The maxillary artery is one of the 2 terminal branches of the external carotid artery. It supplies blood to the deep face and infratemporal fossa. It is not going to the tongue. As far as the nerves go... The lingual nerve is a branch of V3, the mandibular division of the trigeminal nerve. It supplies general sensation (touch and temperature) to the anterior 2/3 of the tongue. (Taste sensation for the anterior 2/3 of the tongue arrives at the tongue by way of the lingual nerve, but the original fibers for taste came from the chorda tympani, a branch of CN VII.) The hypoglossal nerve (CN XII) provides motor innervation to the muscles of the tongue. The glossopharyngeal nerve (CN IX) provides taste sensation and general sensation to the posterior 1/3 of the tongue.

The contents of the paralingual space do NOT include the: Hypoglossal nerve Lingual artery Lingual nerve Submandibular gland Sublingual gland

Lingual artery The paralingual space is a space inside the floor of the mouth. It is bounded by the mylohyoid muscle, the lateral tongue, the hyoid bone, and the oral mucosa. The paralingual space contains the deep portion of the submandibular gland, the lingual nerve and submandibular ganglion, the sublingual gland and fold, and the hypoglossal nerve. The lingual artery is not in the paralingual space.

In reducing an ankylosis of the TMJ, a surgeon provoked an intense hemorrhage by lacerating the artery coursing transversely just medial to the neck of the condyle. Which artery was involved in the accident? Buccal External carotid Maxillary Middle meningeal Superficial temporal

Maxillary The maxillary artery is one of the terminal branches of the external carotid artery which is closely related with the TMJ. It travels medial to the neck of the condyle, so it would be the artery damaged in this scenario. The buccal artery is a branch of the maxillary artery that travels in the cheek. It supplies blood to the cheek mucosa and skin. The external carotid artery is the source of the maxillary artery, but it is not the artery lying medial to the neck of the mandibular condyle. The middle meningeal artery is a branch of the maxillary artery which is deep to the lateral pterygoid muscle. It supplies the calvaria and the dura surrounding the brain. Finally, the superficial temporal artery is the other terminal branch of the external carotid artery. It courses posterior and lateral to the head of the mandible to supply the scalp of the lateral side of the head and lateral face.

The surgical removal of a metastatic tumor in the infratemporal fossa caused an intense hemorrhage. The surgeon clamped the main source of arterial supply to the area, which is the: Internal carotid Lingual Maxillary Posterior auricular Superficial temporal

Maxillary artery The maxillary artery is one of the terminal branches of the external carotid artery. It enters the infratemporal fossa and is the major source of blood for that region. The internal carotid artery is the primary blood supply to the brain. The lingual artery is another branch of the external carotid artery--it supplies blood to the tongue and the floor of the mouth. The posterior auricular is a branch of the external carotid artery that supplies the external ear, scalp and the deeper structures posterior to the ear. The superficial temporal artery is the other terminal branch of the external carotid artery -- it supplies blood to the scalp of the lateral side of the head and the lateral face.

While recovering from multiple dental extractions, an elderly man experienced a radiating pain affecting the lower eyelid, lateral side of the nose, upper lip and over the zygomatic and temporal areas on the left side. Which nerve is involved in the patient's perception of pain? Facial Opthalmic division of trigeminal Glossopharyngeal Mandibular division of trigeminal Maxillary division of trigeminal

Maxillary division of trigeminal The trigeminal nerve is the nerve that supplies sensory innervation to the skin of the face. It has 3 divisions. The maxillary division of trigeminal (V2) is the one that's important for this case--it is a sensory branch of the trigeminal that provides innervation to the skin of the cheek, upper lip, lower eyelid, and the lateral portion of the nose. This is exactly the area that the patient feels pain, so it is the correct answer. The ophthalmic division (V1) is a sensory nerve that passes through the superior orbital fissure and supplies sensory innervation to the eyeball, conjunctiva, nasal mucosa, medial portion of the nose, upper eyelid, forehead, and scalp. The mandibular division (V3) is a sensory and motor nerve--it supplies skin of the lower lip, chin and lower jaw. V3 also provides motor innervation to the muscles of mastication, tensor veli palatini, mylohyoid, the anterior belly of the digastric, and tensor tympani. The facial nerve innervates the muscles of facial expression, but it does not provide sensory innervation to the skin of the face. The glossopharyngeal nerve provides sensory innervation to the pharynx and sensory and taste innervation to the posterior 1/3 of the tongue. But, it does not innervate any skin on the face.

A 45-year old woman with recurrent left middle ear infection (otitis media) complained of partial dryness of her mouth to her ENT surgeon. Taste sensation and hearing were normal. After a thorough clinical examination at the hospital, the doctor concluded that the infection must have spread to a component of the glossopharyngeal nerve (CN IX) that supplies the parotid gland. On which of the following walls of the middle ear is this nerve component located? Anterior Lateral Medial Posterior Roof

Medial To answer this question, you first need to identify the component of the glossopharyngeal nerve that innervates parotid gland. Remember--preganglionic parasympathetic fibers from CN IX are arriving at the otic ganglion via the lesser petrosal nerve and synapsing in the ganglion; the postganglionic fibers then travel on the auriculotemporal nerve (V3) and innervate the parotid gland. So, the lesser petrosal nerve, or fibers that create the lesser petrosal nerve, must have been injured. Now, you just need to think about where these fibers are in the ear. These fibers from the glossopharyngeal nerve are covering the promontory on the medial wall of the ear. The tympanic nerve is a branch of the glossopharyngeal nerve that carries the preganglionic parasympathetic fibers that will eventually travel to the otic ganglion. The tympanic nerve lies on the promontory and creates the tympanic plexus, which gives rise to the lesser petrosal nerve. Given the clinical presentation, the patient must have an infection in the tympanic nerve, tympanic plexus or lesser petrosal nerve. And, all of these nerves are associated with the medial wall of the middle ear.

The parotid space contains all EXCEPT: External carotid artery Facial nerve Intraparotid lymph nodes Medial pterygoid muscle Retromandibular vein

Medial pterygoid muscle The medial pterygoid muscle is not in the parotid space. It serves as the anterior boundary of the parotid fossa. The other structures mentioned are all found within the parotid gland. The facial nerve, retromandibular vein, and external carotid artery all course through the parotid gland. From superficial to deep, they are arranged nerve, vein, artery. The intraparotid lymph nodes are found in the parotid gland also.

Which of the following suprahyoid muscles would be paralyzed if the inferior alveolar nerve were severed at its origin? Geniohyoid m. Hyoglossus m. Mylohyoid m. Stylohyoid m.

Mylohyoid Mylohyoid is innervated by the nerve to mylohyoid, which is a branch of the inferior alveolar nerve. The inferior alveolar nerve is a branch of the mandibular division of the trigeminal nerve. Geniohyoid is a muscle that spans from the mental spines of the mandible to the body of the hyoid bone--it elevates the hyoid and depresses the mandible. It is innervated by the ventral primary ramus of spinal nerve C1 via fibers carried by the hypoglossal nerve. Hyoglossus is a tongue muscle; it is innervated by the hypoglossal nerve (CN XII). Finally, stylohyoid is a muscle that spans from the styloid process to the hyoid bone. It elevates and retracts the hyoid, and it is innervated by the facial nerve.

What might happen if chorda tympani was injured?

No taste sensation to the anterior 2/3 of the tongue and no secretomotor innervation to the sublingual and submandibular glands!

A 35-year-old man was admitted to the hospital complaining of double vision (diplopia), inability to see close objects, and blurred vision in the right eye. A vertebrobasilar angiogram revealed an aneurysm of the superior cerebellar artery close to its origin on the right side. The doctor attributed the symptoms to the compression of an adjacent cranial nerve by the aneurysm. The compressed nerve is the: Abducens (CN VI) Oculomotor (CN III) Optic (CN II) Trigeminal (CN V) Trochlear (CN IV)

Oculomotor (CN III) Given the patient's symptoms, it seems that some nerve involving vision and the ability to control the eye has been injured. Now, you need to think about which nerve might be damaged by an aneurysm of the superior cerebellar artery. The oculomotor nerve, which innervates the superior rectus, medial rectus, inferior rectus, and inferior oblique muscles, passes between the posterior cerebral artery and the superior cerebellar artery. It could be injured if there was enlargement of or damage to either of these vessels. None of the other cranial nerves are in the right position to be injured from an aneurysm of the superior cerebellar artery.

These structures are all located in the superficial fascia of the neck EXCEPT External jugular vein Platysma muscle Cervical branch of the facial nerve Omohyoid muscle Transverse cervical nerve

Omohyoid muscle The omohyoid muscle is a muscle found in the anterior and posterior triangle of the neck. It depresses and stabilizes the hyoid bone. Like all muscles (except the muscles of facial expression), the omohyoid is not found in the superficial fascia. The external jugular vein is a small vein found in the superficial tissue on the lateral side of the neck. The platysma is a muscle of facial expression that draws down the corners of the mouth and depresses the mandible. It inserts into the skin, so it is found in the superficial fascia. The cervical branch of the facial nerve, which innervates the platysma, is also in the superficial fascia. Finally, the transverse cervical nerve is a branch of the cervical plexus that innervates the skin of the anterior neck. Because it is providing cutaneous innervation, it is found in the superficial fascia.

An adolescent boy suffers from severe acne. As is often the case he frequently squeezed the pimples on his face. He subsequently develops a fever and deteriorates into a confused mental state and drowsiness. He is taken to his physician and after several tests a diagnosis of cavernous sinus infection and thrombosis is made. The route of entry to the cavernous sinus from the face was most likely the: Carotid artery Mastoid emissary vein Middle meningeal artery Ophthalmic vein Parietal emissary vein

Ophthalmic vein The ophthalmic veins are continuous with the facial vein and the pterygoid plexus of veins. These veins drain the face toward the cavernous sinus. They are valveless, so infections from the face can drain into the cavernous sinus. Besides causing fever and confusion, thrombotic congestion and edema in the cavernous sinus can compress the nerves that traverse that space to exit through the superior orbital fissure(CN III, CN IV, CN V1, and CN VI). This can affect the function of the ocular muscles, so one symptom of a cavernous sinus infection might be an inability to perform different eye movements.

Inability to close the lips relates to the action of which muscle? Anterior belly of the digastric Mylohyoid Orbicularis oris Platysma Zygomaticus major

Orbicularis oris Orbicularis oris is a muscle of facial expression. It surrounds the lips, and allows for pursing of the lips. It is innervated by the buccal branch of the facial nerve. The anterior belly of the digastric and mylohyoid both help elevate the hyoid bone and depress the mandible. They are both innervated by a branch of the mandibular division of the trigeminal nerve (V3). Platysma and zygomaticus major are both muscles of facial expression, innervated by the facial nerve. Platysma draws the corners of the mouth down and aids in depressing the mandible; zygomaticus major elevates and draws the corners of the mouth laterally.

A muscle that purses the lip is the: A. Levator labii superioris. B. Orbicularis oculi. C. Orbicularis oris. D. Depressor anguli oris. E. Procerus.

Orbicularis oris.

During a hysterectomy, the uterine vessels are ligated. However, the patient's uterus continues to bleed. The most likely source of blood still supplying the uterus is from which artery? Inferior vesical Internal pudendal Middle rectal Ovarian Superior vesical

Ovarian The ovarian artery has branches which supply the uterus. In fact, this artery anastomoses with the uterine artery. So, if the uterus is still bleeding after ligating the uterine artery, the ovarian artery is probably supplying the uterus. The inferior vesical artery supplies the inferior part of the bladder--it anastomoses with the middle rectal artery. The internal pudendal artery supplies blood to the perineum. The middle rectal artery supplies blood to the rectum. The superior vesical artery supplies blood to the superior bladder.

Cutting of the hypoglossal nerve in the hypoglossal canal would not interrupt the nerve supply to the: Hyoglossus muscle Genioglossus muscle Palatoglossus muscle Styloglossus muscle

Palatoglossus muscle All of the tongue muscles are innervated by the hypoglossal nerve. So, hyoglossus, genioglossus, and styloglossus are all innervated by the hypoglossal nerve. Although palatoglossus sounds like a tongue muscle, it is a palatal muscle, innervated by the vagus nerve. (Palatoglossus is the only "glossus" muscle that is not innervated by the hypoglossal nerve.)

Which structure forms a border of the tonsillar fossa? Torus tubarius Palatopharyngeal fold Salpingopharyngeal fold Lateral glossoepiglottic fold Medial pterygoid plate

Palatopharyngeal fold

During a prostatectomy, the surgeon attempts to protect the prostatic plexus of nerves which contains nerve fibers that innervate penile tissue to cause erection. From which nerves do these fibers originate? Deep perineal Dorsal nerve of the penis Genitofemoral Pelvic splanchnics Pudendal

Pelvic splanchnics Erection is mediated by parasympathetic nerves, and the pelvic splanchnic nerves are the parasympathetic nerves that innervate the smooth muscle and glands of all pelvic viscera. So, the pelvic splanchnic nerves are the nerves contributing the fibers to the prostatic plexus which innervate penile/clitoral erectile tissue to cause erection.

A first year resident attempts to insert a central venous catheter into the heart by a subclavicular approach to the subclavian vein. Following the attempt, it is noted that the patient has difficulty breathing. What nervous structure lying immediately deep to the subclavian vein, as it crosses the anterior scalene muscle, may have been injured? External branch of the superior laryngeal nerve Phrenic nerve Recurrent laryngeal nerve Sympathetic trunk Vagus nerve

Phrenic nerve The phrenic nerve is formed from contributions from the C3, 4, and 5 ventral primary rami. It lies on the anterior scalene muscle, immediately deep to the subclavian vein. Since the phrenic nerve innervates the diaphragm, the patient's symptom (difficulty breathing) fits with an injury to the phrenic nerve. The external branch of the superior laryngeal nerve travels with the superior thyroid artery to innervate the cricothyroid muscle. It is found medial to the scalene muscles. The recurrent laryngeal nerves are branches of the vagus that branch in the thorax. The right recurrent laryngeal loops around the subclavian artery and the left recurrent laryngeal loops around the aortic arch. Then, the two nerves ascend to the larynx, running medial to the scalenes. The sympathetic trunk lies in the prevertebral fascia behind the carotid sheath and in front of the prevertebral muscles. Finally, the vagus nerve runs in the carotid sheath, superficial to the scalene muscles.

A 27-year-old man who is a professional weight lifter comes to his physician complaining of recent weakness in his left arm and frequent tingling in his hand and fingers during exercise sessions which subsides with rest. He is diagnosed as having vasuclar insufficiency due to scalenus anticus syndrome and as a remedy it is decided to transect the anterior scalene muscle where it inserts on the first rib. During surgery, which structure in contact with the anterior surface of the muscle must the surgeon be careful of sparing? Inferior trunk of the brachial plexus Long thoracic nerve Phrenic nerve Sympathetic trunk Vagus nerve

Phrenic nerve The phrenic nerve is formed from contributions from the C3, 4, and 5 ventral primary rami. It lies on the anterior scalene muscle, which means that the surgeon would need to watch for this nerve while transecting the anterior scalene. None of the other structures lie upon the anterior scalene. The roots of the brachial plexus come out between the anterior and middle scalenes, and the trunks lie lateral to the anterior scalene. The long thoracic nerve comes off the C5, 6 and 7 roots of the brachial plexus and can be seen piercing the middle scalene muscle. This nerve lies on the surface of serratus anterior in the thorax, innervating that muscle. The sympathetic trunk is found deep to the scalene muscles; it runs in the prevertebral fascia along the front of the vertebral column. The vagus nerve runs within the carotid sheath, superficial to the anterior scalene muscle. It passes between the subclavian artery and vein.

The cervical plexus innervates all of the following structures EXCEPT: Diaphragm Omohyoid muscle Platysma Skin over thyroid cartilage Skin over clavicle

Platysma Platysma is a muscle of facial expression. It inserts into the skin and helps lower the angle of the mouth and depress the mandible. It is innervated by the cervical branch of the facial nerve (CN VII). All of the other muscles or areas of skin are innervated by branches of the cervical plexus, a plexus made of the ventral primary rami of the C1-C4 spinal nerves. The diaphragm is innervated by the phrenic nerve, which is made from the C3, C4 and C5 ventral primary rami. The omohyoid muscle is innervated by ansa cervicalis, a motor branch of the cervical plexus. The skin over the thyroid cartilage is innervated by the transverse cervical nerve, a cutaneous sensory nerve from C2 and C3 roots of the cervical plexus. The skin over the clavicle is innervated by other cutaneous sensory nerves from the C3 and C4 roots of the cervical plexus--the supraclavicular nerves.

The muscle most responsible for the abduction of the vocal folds is the : Arytenoid Cricothyroid Lateral cricoarytenoid Posterior cricoarytenoid Thyroarytenoid

Posterior cricoarytenoid The posterior cricoarytenoid is the only muscle that abducts the vocal folds. This muscle is innervated by the inferior laryngeal nerve, which is a continuation of the recurrent laryngeal nerve. If this muscle is denervated, the vocal folds may be paralyzed in an adducted position, which would prevent air from entering the trachea. Arytenoid, lateral cricoarytenoid, and thyroarytenoid all adduct the vocal folds. Cricothyroid is the only laryngeal muscle innervated by the external branch of the superior laryngeal. It tenses the vocal ligaments by tipping the thyroid cartilage forward relative to the cricoid cartilage.

Twenty-four hours following a partial thyroidectomy where the inferior thyroid artery was also ligated (tied off), the patient now spoke with a hoarse voice (whisper), and had difficulty in breathing. Which nerve was injured? Internal branch of superior laryngeal Ansa cervicalis Ansa subclavia Recurrent laryngeal External branch of superior laryngeal

Recurrent laryngeal The recurrent laryngeal nerve runs with the inferior thyroid artery toward the lower lobes of the thyroid. This means that the recurrent laryngeal nerve would be at risk in any surgery involving the inferior thyroid artery or the inferior poles of the thyroid. The recurrent laryngeal nerve becomes the inferior laryngeal nerve at the inferior border of cricoid cartilage, and this nerve continues on to innervate all the muscles of the larynx with the exception of cricothyroid. So, an injury to the recurrent laryngeal nerve might lead to hoarseness and difficulty breathing (due to a laryngeal spasm). The internal branch of superior laryngeal runs with the superior laryngeal artery and pierces the thyrohyoid membrane. Ansa cervicalis is a branch of the cervical plexus which hangs in front of the internal jugular vein. It innervates the strap muscles, not the laryngeal muscles. Ansa subclavia is part of the sympathetic trunk which loops around the subclavian artery. Finally, the external branch of the superior laryngeal nerve runs with the superior thyroid artery. This is the artery and nerve that might be damaged when removing the superior lobe of the thyroid.

In repairing a damaged right subclavian artery, the surgeon notices and protects a large nerve passing around to the posterior surface of the artery. This nerve, which does not encircle the subclavian on the left side, is the: Phrenic Vagus Recurrent laryngeal Sympathetic trunk Ansa cervicalis

Recurrent laryngeal The recurrent laryngeal nerves have different relationships to structures on the right and left sides of the body. On the right side, the recurrent laryngeal nerve loops around the right subclavian artery at approximately the T1/T2 vertebral level. On the left side, the recurrent laryngeal nerve loops around the arch of the aorta, behind the ligamentum arteriosum, at approximately the T4/T5 level. After this, both the right and left recurrent laryngeals ascend in the tracheoesophageal groove to innervate all the intrinsic muscles of the larynx, except the cricothyroid. So, you should see that the right recurrent laryngeal has a special relationship to the right subclavian that would put it in danger during a surgical procedure. The phrenic nerves lie on the anterior scalene muscles on both the right and left side. This means that the phrenic nerves are anterior to the subclavian arteries. The vagus nerves also pass anterior to the subclavian arteries. On both the left and right side, the sympathetic trunk lies in front of the prevertebral muscles in the prevertebral fascia. Finally, the ansa cervicalis hangs in a loop anterior to the internal jugular vein on the right and left sides. None of these other structures have special relationships with the right subclavian artery.

Exploration of the tracheoesophageal groove at the level of the thyroid gland would reveal what important structure bilaterally? Cervical sympathetic trunk Phrenic nerve Pyramidal lobe Recurrent laryngeal nerve Superior thyroid artery

Recurrent laryngeal nerve The left and right recurrent laryngeal nerves can be found in the left and right tracheoesophageal grooves. This is a very characteristic relationship that you need to know! The cervical sympathetic trunk is much deeper in the neck--it lies in the prevertebral fascia, in front of the prevertebral muscles. The phrenic nerve lies on the surface of the anterior scalene muscles before it descends into the thorax to innervate the diaphragm. The pyramidal lobe of the thyroid gland is a small tract of tissue that may extend upward from the isthmus of the thyroid. This is a developmental remnant that marks the thyroid's route of migration from the foramen cecum of the tongue. The superior thyroid artery is a branch of the external carotid artery--it travels to the superior lobes of the thyroid and runs with the external branch of the superior laryngeal nerve.

Which muscle is also known as the sphenomeniscus? Inferior head of the lateral pterygoid Masseter Medial pterygoid Superior head of the lateral pterygoid Temporalis

Superior head of the lateral pterygoid The superior head of lateral pterygoid is sometimes called sphenomeniscus due to its insertion into the disc of the temporomandibular joint. Remember, the superior head of the lateral pterygoid inserts into the disc of the TMJ, while the inferior head of the lateral pterygoid inserts into the neck of the mandible (pterygoid fovea). None of the other listed muscles insert into a joint like the superior head of the lateral pterygoid.

Repeated middle ear infections have destroyed the tympanic plexus in the middle ear cavity. The loss of preganglionic parasympathetic fibers that pass through the plexus diminish production of: Mucus in the nasal cavity Mucus on the soft palate Saliva by the parotid gland Saliva by the submandibular and sublingual glands Tears by the lacrimal gland

Saliva by the parotid gland The tympanic plexus is formed by the tympanic nerve, a branch of the glossopharyngeal nerve (CN IX). The tympanic plexus provides sensory innervation to the mucosal lining of the middle ear. The lesser petrosal nerve also comes out of the tympanic plexus. This nerve carries preganglionic parasympathetic fibers from the tympanic plexus - these fibers eventually synapse in the otic ganglion. The postsynaptic fibers that leave the otic ganglion provide parasympathetic secretomotor innervation to the parotid gland. So, if the tympanic plexus was destroyed, the lesser petrosal nerve would be destroyed and the parotid gland would not secrete saliva.

Which pair of structures does NOT differentiate from comparable embryonic structures in the male and female? Bulb of corpus spongeosum and vestibular bulb Shaft of penis and labia majora Glans of penis and glans of clitoris Crus of corpus cavernosum penis and crus of corpus cavernosum clitoris

Shaft of penis and labia majora The shaft of the penis is an analog of the shaft of the clitoris, while the labia majora is derived from the same embryonic structures as the scrotum. The other three answer choices list structures that come from comparable embryonic structures in the male and female

To drill a mandibular tooth without causing undue pain, a dentist has injected an anesthetic into the space located between the medial pterygoid muscle and the mandible near the lingula. Given the nerves passing through the immediate vicinity of the injection site, where would one expect anesthesia in addition to the mandibular teeth? back of tongue external ear maxillary incisor teeth skin of chin upper lip

Skin of chin The dentist would need to numb the inferior alveolar nerve, which is the nerve that runs in the mandibular foramen and provides sensory innervation to the teeth. The inferior alveolar nerve also gives off the mental nerve, which is a sensory nerve innervating the skin of the chin. So, if the inferior alveolar nerve was anesthetized, the mental nerve would be anesthetized, too. Then, the patient would lose sensory innervation to the skin of the chin. The back of the tongue (posterior 1/3) receives sensory innervation from the glossopharyngeal nerve. This nerve would not be exposed to the anesthetic. The external ear is innervated by the auriculotemporal nerve, which is not near the area of anesthesia. The maxillary incisor teeth are innervated by the superior alveolar nerves, which are branches of the maxillary division of the trigeminal nerve (V2). The skin of the upper lip is also innervated by a branch of V2--the infraorbital nerve. These branches of V2 would not be exposed to the anesthesia. If too much anesthesic was injected or if the parotid fascia was pierced by the needle, it would be possible for the anesthesic to diffuse through the soft tissue and paralyze the facial nerve. A high dose of anesthesic might also diffuse to paralyze the muscles of mastication.

Which structure or area receives NO nerve fibers from the cervical plexus? Diaphragm Skin over the angle of the mandible Skin over the external occipital protuberance Thyrohyoid muscle

Skin over the external occipital protuberance The skin over the external occipital protuberance is innervated by the greater occipital nerve, which is from the dorsal primary ramus of C2. Since the cervical plexus is made of the ventral primary rami of C1 through C4, the greater occipital nerve has no connection with the cervical plexus. The diaphragm is innervated by the phrenic nerve, which receives contributions from the ventral primary rami of C3, C4, and C5. The skin over the angle of the jaw is innervated by the great auricular nerve, which comes from C2 and C3 branches of the cervical plexus. Finally, the thyrohyoid muscle is innervated by the ansa cervicalis (specifically, C1 and C2 fibers carried by hypoglossal nerve), a motor component of the cervical plexus that innervates the strap muscles.

A patient with a facial nerve paralysis suffers from inability to dampen loud noises (hyperacusis) due to denervation of which muscle? Posterior belly of digastric Stapedius Tensor tympani Stylohyoid muscle

Stapedius Stapedius is a small muscle in the ear innervated by the facial nerve. It dampens large vibrations of the stapes and the tympanic membrane; this allows the ear to diminish loud noises. If the facial nerve is paralyzed (as seen with Bell's palsy), the nerve to stapedius is lost, and the ear cannot lessen the vibrations of stapedius. This causes hyperacusis.

A six-year-old child, whose medical history includes a rather difficult birth, has a permanently tilted head posture, with the right ear near the right shoulder and the face turned upward and to the left. Which of the following muscles was very likely damaged during birth? Anterior scalene Omohyoid Sternocleidomastoid Trapezius Platysma

Sternocleidomastoid If the sternocleidomastoid is damaged at birth and becomes shortened, a patient may have muscular torticollis. The neck will appear twisted and the chin elevated because the sternocleidomastoid muscle will be too short on one side. This patient has an injured right sternocleidomastoid, since his head is turned down on the right side where the sternocleidomastoid is too short. Patients may also have spasmotic torticollis, where the neck is twisted and elevated due to a temporary spasm of the sternocleidomastoid muscle. None of the other muscles listed would cause similar symptoms if they were injured. It's important to remember the association between torticollis and the sternocleidomastoid!

During childbirth, an excessive anteroposterior compression of the head may tear the anterior attachment of the falx cerebri from the tentorium cerebelli. The bleeding that follows is likely to be from which of the following venous sinuses? Occipital sinus Sigmoid sinus Straight sinus Superior sagittal sinus Transverse sinus

Straight sinus The straight sinus drains the deep cerebrum--it lies within the junction of the falx cerebri and tentorium cerebelli. This is exactly the location that was damaged in birth, so this is the correct answer. The occipital sinus drains the cerebellum--it lies within the dura mater at base of falx cerebelli. It is inferior to the straight sinus. The sigmoid sinus drains the blood from the brain into the internal jugular vein--it lies within sigmoid groove, covered by dura mater. The superior sagittal sinus drains the cerebral hemispheres--it lies superiorly within falx cerebri, near the superior border of the skull. The transverse sinus lies within the attachment of tentorium cerebelli to the inner surface of the calvaria.

A muscular patient who regularly lifts weights presents with pain and weakness in his right upper limb that has been slowly developing over time. His limb is cool and there is an obvious vascular insufficiency in his upper extremity. Subsequent tests show that a large vessel passing between the anterior and middle scalene muscles is being occluded by hypertrophy (enlargement) of the muscles due to the excercise. The artery involved is the: axillary brachial brachiocephalic subclavian suprascapular

Subclavian The subclavian artery runs between the anterior and middle scalene muscles. Since this is the major artery supplying blood to the upper limb, occluding this artery might cause the symptoms of pain and weakness. This patient has a thoracic outlet syndrome, which means that one of the arteries or nerves that is passing from the thoracic outlet into the upper limb or or lower neck is being compressed. Thoracic outlet syndrome may be caused by several things, including a cervical rib (when ribs start forming on C7, not T1) , hypertrophy of scalene muscles, or hyperabduction of the upper limb. Specifically, this patient has scalenus anticus syndrome, which means that there is a compression of structures between the scalene muscles.

Which structure lies immediately anterior to the right anterior scalene muscle at its costal attachment? Subclavian artery Subclavian vein Thoracic duct Thyrocervical trunk Vagus nerve

Subclavian vein The subclavian vein lies anterior to the right and left anterior scalene muscles. It can be found coursing over the anterior scalene muscles on both the right and left sides. The subclavian arteries lie posterior to the anterior scalene muscles--they travel between the anterior and middle scalenes on both sides of the neck. The thoracic duct is found on the left side only--it would not be on the right side of the neck. On the left side, the thoracic duct enters the left brachiocephalic vein where it is formed by the union of the subclavian and internal jugular veins. So, the thoracic duct does contact the anterior scalene muscle, but only on the left side of the neck. The thyrocervical trunk is a branch of the first part of the subclavian artery, medial to the anterior scalene. It is not in contact with the anterior scalene. Finally, the vagus nerve travels with the carotid vessels in the carotid sheath. It is anterior to the anterior scalene, but not in immediate contact with the muscle. This means that subclavian vein is the best answer.

All of the following may be found in the paralingual space EXCEPT: Hypoglossal nerve Lingual nerve Sublingual gland Submandibular gland duct Superficial lobe of the submandibular gland

Superficial lobe of the submandibular gland The paralingual space is found in the floor of the mouth, deep to the mylohyoid muscle. The space is bounded by the lateral tongue, the hyoid bone, and the oral mucosa. Remember--the submandibular gland has a deep and superficial portion. The deep portion of the submandibular gland is found deep to the mylohyoid muscle, while the superficial portion of the submandibular gland is found superficial to the mylohyoid muscle. So, the deep portion of the submandibular gland is in the paralingual space, while the superficial portion of the submandibular gland is not. See Netter Plate 55 for a picture of this. The paralingual space also contains the submandibular duct and sublingual caruncle, the lingual nerve and submandibular ganglion, the hypoglossal nerve, and the sublingual gland and fold.

Starting from a position gazing straight ahead, to direct the gaze downward, the inferior rectus muscle must be active along with the: Superior oblique Inferior oblique Medial rectus Lateral rectus Superior rectus

Superior Oblique The inferior rectus muscle depresses the eye and medially rotates it. So, to direct the gaze downward, you want to find a muscle that will depress the eye while counterbalancing the medial rotation with lateral rotation. And, the superior oblique, innervated by the trochlear nerve (CN IV), does just that--it depresses the eye while laterally rotating it. The inferior oblique muscle laterally rotates the eye and elevates the eye. The medial rectus adducts the eye--it does not raise or lower the eye. The lateral rectus abducts the eye--it also does not raise or lower the eye. Finally, the superior rectus elevates the eye and draws it medially.

Any irritation of the diaphragm (e.g. infection, tumor) may create referred pain that seems (to the patient) to originate in the area atop the shoulder. This is due to the fact that the phrenic nerve shares spinal segments with what cutaneous nerve in the shoulder region? Dorsal scapular Lesser occipital nerve Long thoracic nerve Supraclavicular nerve Suprascapular nerve

Supraclavicular nerve The phrenic nerve consists of contributions from the ventral primary rami of C3, C4, and C5. The supraclavicular nerves come from the C3 and C4 roots of the cervical plexus. So, these nerves share spinal segments. This explains why pain from the diaphragm is referred to the area where the supraclavicular nerves provide cutaneous innervation. The lesser occipital nerve is another cutaneous nerve from the cervical plexus, but it is mostly made of fibers from the C2 ventral rami. The other nerves listed are all motor nerves from the brachial plexus. The dorsal scapular nerve comes from the C5 ventral primary ramus-- it innervates rhomboideus major and minor and the lower portion of levator scapulae. The long thoracic nerve comes from C5, C6, and C7 ventral primary rami--it innervates serratus anterior. The suprascapular nerve comes from the superior trunk of the brachial plexus--it innervates supraspinatus and infraspinatus.

Paralysis of which of the following muscles would impede retraction of the mandible? Buccinator Lateral pterygoid, lower portion Lateral pterygoid, upper (sphenomeniscus) portion Medial pterygoid Temporalis

Temporalis Temporalis is the important muscle for retracting the mandible! Buccinator is a muscle on the side of the face which pulls the corner of mouth laterally and presses the cheek against the teeth. Both portions of the lateral pterygoid protract the mandible and open the mandible. (Remember: lateral pterygoid is the only muscle that opens the mandible!) Finally, the medial pterygoid protracts and elevates the mandible.

All of the following are true about the middle ear EXCEPT: The joints between ossicles are synovial The chorda tympani nerve is related to the lateral wall The facial nerve passes in a canal situated in the medial and anterior walls The auditory tube connects the nasopharynx with the anterior wall Its mucous membrane is supplied by the glossopharyngeal nerve (CN IX)

The facial nerve passes in a canal situated in the medial and anterior wall. The facial nerve passes in a canal situated in the posterior and medial walls of the middle ear. It is not associated with the anterior wall. The other 4 statements are true. The ossicles articulate with each other at synovial joints. The chorda tympani is a branch of the facial nerve that is related to the lateral wall of the middle ear, near the tympanic membrane. It passes between the malleus and incus. The auditory tube is found in the anterior wall--it is a path of communication between the nasal portion of the pharynx and the tympanic cavity that allows pressure to equalize on either side of the tympanic membrane. Finally, the mucous membrane of the middle ear is supplied by branches of the tympanic plexus, which is formed by the tympanic branch of the glossopharyngeal nerve.

Which of the following is true of the inferior thyroid arteries? They arise from the external carotid arteries They cross over the superior cervical sympathetic ganglion They supply most of the anterior surface of the thyroid gland They often supply all four parathyroid glands

They often supply all four parathyroid glands The inferior thyroid arteries are branches of the thyrocervical trunk. (The superior thyroid arteries are branches of the external carotid artery.) The inferior thyroid arteries are closely associated with the middle cervical sympathetic ganglia and supply the inferior poles of the thyroid. They are found on the posterior surface of the thyroid gland, which is where the parathyroid glands are located. So, they are the primary source of blood for the four parathyroid glands.

A 50-year-old woman is brought to the operating room for a biopsy of a suspicious looking nevus (mole) which is found along the anterior border of her sternocleidomastoid muscle at about its midlength. What nerve must be locally anesthetized for the operation to proceed without pain? Ansa cervicalis External branch of the superior laryngeal Great auricular Supraclavicular Transverse cervical

Transverse cervical To answer this question, you want to figure out which nerves supply sensory innervation to the skin of the anterior neck. The transverse cervical nerves, which come off the C2 and C3 roots of the cervical plexus, are the nerves that supply the skin of the neck anteriorly. So, these nerves would need to be anesthetized. The great auricular and supraclavicular nerves are also cutaneous sensory nerves from the cervical plexus. The great auricular nerve innervates the skin of the ear and the area below the ear. The supraclavicular nerves innervate the skin at the root of the neck and the upper chest and upper shoulder.

Frey's Syndrome is marked by profuse sweating over one cheek, temple, and surrounding areas of the face, precipitated by eating. The condition may be idiopathic, but often follows parotid surgery. The condition is attributable to abberant reinnervation, the redirection of autonomic fibers normally going to salivary glands being redirected to sweat glands. What is the source of the nerve fibers involved? Facial Glossopharyngeal Oculomotor Trigeminal Vagus

Trigeminal Frey's syndrome is a condition in which the postganglionic parasympathetic nerves that are contained in the auriculotemporal nerve (which normally supply the parotid gland) are redirected toward the sweat glands overlying the parotid gland. This means that a patient with Frey's syndrome sweats in the area over the parotid gland while eating. Since the auriculotemporal nerve is a branch of V3, the nerve fibers involved in Frey's syndrome are from the trigeminal nerve. The nerve fibers on the external carotid and internal carotid arteries are sympathetic fibers. Remember--sympathetic fibers create periarterial plexuses that travel with the vasculature to reach different targets around the body. However, the parotid gland is innervated parasympathetically, not sympathetically. The glossopharyngeal nerve contributes to the innervation of the parotid gland by supplying preganglionic parasympathetic fibers to the otic ganglia, by way of the lesser petrosal nerve. However, these fibers are not involved with Frey's syndrome--this condition involves the misdirection of the postganglionic parasympathetic fibers. Finally, the vagus is not involved with innervating the parotid gland.

If a person looking inward towards their nose is unable to look down, which nerve may be injured? Abducens (CN VI) Inferior division of oculomotor (III) Optic (II) Superior division of oculomotor (III) Trochlear (IV)

Trochlear (IV) Since the actions of the extraocular muscles are complex, it is necessary to turn the eye to a position where a single action of each muscle predominates. To isolate the superior and inferior recti, the patient needs to turn the eye outward by approximately 25 degrees. This places the superior rectus in position to raise the eye and the inferior rectus in position to lower the eye. Turning the eye inward approximately 50 degrees places the inferior oblique in position to raise the eye and the superior oblique in position to lower the eye. The medial and lateral recti are the easy muscles -- they may be checked while the eye is staring straight ahead since they have simple planar actions So, this patient is looking inward, which means that the obliques are being tested. The patient can't look downward, which shows that the superior oblique is not functional. This is the only muscle innervated by the trochlear nerve (CN IV). Abducens (CN VI) innervates the lateral rectus muscle, which is tested by asking the patient to move the eye outward. The inferior division of the oculomotor nerve innervates inferior rectus, inferior oblique, and medial rectus. The superior branch of the oculomotor nerve innervates levator palpebrae superioris and superior rectus muscles. Finally, the optic nerve (CN II) provides the special sense of vision, and it is not tested in the eye-movement tests.

A 60-year-old man has occasional blackouts and light-headedness. Studies reveal atherosclerotic plaques within the common carotid arteries and the bifurcation of the vessels. A carotid endartectomy is undertaken. Which nerve bundle running vertically within the carotid sheath must the surgeon be careful not to injure? Accessory Cervical sympathetic trunk Glossopharyngeal Hypoglossal Vagus

Vagus In a carotid endarterectomy, surgeons enter the carotid artery and excise the diseased endothelium and media of the artery so that the lumen is left with a smooth lining. To do this, the surgeons must enter the carotid sheath, which means that any structure within that sheath or near that sheath might be injured. This question specifically asks which structure in the sheath could be injured, so the correct answer is the vagus nerve. The carotid sheath contains 2 nerves: the vagus and the superior ramus of ansa cervicalis. It also contains 3 vessels: internal carotid, common carotid, and internal jugular vein. Any of these structures could be damaged during the procedure. The accessory nerve is most closely associated with the posterior triangle of the neck. It cuts through this triangle to innervate sternocleidomastoid and trapezius and lies posterior to the carotid vessels The cervical sympathetic trunk lies in front of the prevertebral muscles in the prevertebral fascia. The glossopharyngal nerve comes out of the jugular foramen and divides into 2 branches--pharyngeal and lingual branches. This nerve sweeps near the carotid sheath, but it's not in the carotid sheath. The hypoglossal nerve travels lateral to the carotid vessels before entering the floor of the mouth. It crosses the carotid artery, but it's not in the sheath.

A 55-year-old woman has difficulty swallowing and frequently aspirates fluids while drinking. She is diagnosed as having a skull base tumor occupying the space behind the jugular foramen. Involvement of which structure is responsible for the findings? Ansa cervicalis Cervical sympathetic trunk Accessory nerve Hypoglossal nerve Vagus

Vagus The vagus nerve, which exits the skull through the jugular foramen, is the motor nerve to the pharynx. So, it allows for swallowing. This patient's symptoms and the location of the tumor clearly point to an injury of the vagus nerve. The accessory nerve also exits the skull through the jugular foramen--it could be injured from this tumor, but the symptoms are not suggestive of an accessory nerve injury. If her accessory nerve was damaged, the patient's sternocleidomastoid and trapezius would be denervated, which means the acromion of her shoulder would sag. The other nerve that comes through the jugular foramen is the glossopharyngeal nerve--this nerve might also be damaged by the tumor.

The constrictor muscles of the pharynx receive their motor nerve supply from the: Glossopharyngeal nerve Hypoglossal nerve Spinal accessory nerve Sympathetic trunk Vagus nerve

Vagus nerve (CN X) The vagus nerve supplies motor innervation to the muscles of the larynx and pharynx, with the exception of stylopharyngeus (innervated by the glossopharyngeal nerve). It also supplies motor innervation to the palate muscles, with the exception of tensor veli palatini (innervated by the V3 division of the trigeminal nerve). The glossopharyngeal nerve (CN IX) provides the sensory, but not motor, innervation to the pharynx. The hypoglossal nerve (CN XII) provides motor innervation to the muscles of the tongue. The accessory nerve (CN XI) provides motor innervation to the trapezius and the sternocleidomastoid. Finally, the sympathetic trunk supplies sympathetic innervation to the head and neck.

A patient complains of loss of hearing in the right ear. Examination reveals ankylosis (otosclerosis) of the footplate of the stapes to the surrounding bone. Which part of the bony labyrinth is involved? Aditus ad antrum Cochlear (round) window Cochlear duct Internal acoustic meatus Vestibular (oval) window

Vestibular (Oval) Window

The vocalis muscle is most responsible for the fine control of phonation because of its attachment into the: Arytenoid cartilage Cricoid cartilage Thyroid cartilage Vestibular ligament Vocal ligament

Vocal ligament Vocalis represents the innermost fibers of the thyroarytenoid muscle, from the inner surface of the thyroid cartilage. Because vocalis inserts on the vocal ligament, it can relax very specific segments of the vocal ligament to adjust the pitch. Although there are many muscles that insert on the other pieces of cartilage listed, vocalis is the only one that can provide for such fine control of tone because it is inserting right on the vocal ligament.

The facial muscle most responsible for moving the lips both upward and laterally to produce a smile is: Buccinator Levator anguli oris Levator labii superioris Platysma Zygomaticus major

Zygomaticus major Zygomaticus major is innervated by the zygomatic and buccal branches of the facial nerve. It elevates the corner of the mouth and draws it laterally. Remember zygomaticus major as the "smile" muscle! Levator anguli oris is close to the correct answer. This muscle, which is innervated by the facial nerve, elevates the corners of the mouth. However, zygomaticus major also draws the mouth laterally to produce a smile, so this answer is more correct. The three other muscles are all innervated by branches of the facial nerve. Buccinator is innervated by the buccal branches of the facial nerve. It allows the cheek to be pulled taut against the molar teeth. Levator labii superioris is innervated by the buccal branch of the facial nerve--it pulls the lip upwards. Platysma is innervated by the cervical branches of the facial nerve. It draws the corners of the mouth downward and aids in depressing the mandible.

Under normal conditions, fertilization occurs in which part of the female reproductive tract? Infundibulum of the Uterine Tube Ampulla of the Uterine Tube Isthmus of the Uterine Tube Uterine Lumen Cervical Canal

ampulla of the uterine tube Fertilization normally takes place in the ampulla of the uterine tube. This is the middle segment of the uterine tube, and it is the longest and widest segment. The infundibulum is the funnel-shaped distal end of the uterine tube, and the isthmus is the narrowest part of the uterine tube which connects directly to the uterus. The uterine lumen is the site of implantation, not fertilization. Finally, the cervical canal is the pathway out of the uterus at its inferior end-- it would not be an appropriate site for fertilization or implantation.

Which nerve provides motor innervation to the buccinator muscle? Auriculotemporal nerve Buccal branches of VII Buccal nerve Mandibular division of V Marginal mandibular nerve

buccal branches of VII The buccal branches of the facial nerve provide motor innervation to the buccinator muscle. Remember, these buccal branches of the facial nerve are motor nerves only--they do not do any sensory innervation. Don't mix this nerve up with the buccal nerve, which is a branch of the mandibular division of the trigeminal nerve (V3)! The buccal nerve is a sensory nerve only--it does not innervate any muscles; it only gives sensory innervation to the skin of the cheek and the mucosal lining of the cheek. The auriculotemporal nerve is also part of the mandibular division of the trigeminal nerve--it carries the postganglionic parasympathetic fibers to the parotid gland and provides sensory innervation to the skin of the anterosuperior ear, part of the external auditory meatus, and the temporomandibular joint. The marginal mandibular nerve is another branch of the facial nerve--it innervates the muscles of facial expression on the lower lip and chin.

A cranial fracture through the foramen ovale that compresses the enclosed nerve, will have an effect on all muscles EXCEPT : Tensor tympani Masseter Buccinator Mylohyoid Temporalis

buccinator The mandibular division of the trigeminal nerve (V3) is transmitted through foramen ovale. Branches from this nerve innervate the muscles of mastication, like temporalis and masseter. A branch from V3 also innervates tensor tympani, which is a muscle that dampens the vibrations of the tympanic membrane. Mylohyoid is innervated by a branch of the inferior alveolar nerve which is also from V3--this muscle elevates the hyoid bone and the tongue and depresses the mandible. Buccinator is a muscle on the side of the cheek which is innervated by the facial nerve (VII). This muscle, which pulls the corner of mouth laterally and presses the cheek against the teeth, would still be functional even after disrupting the mandibular division of the trigeminal nerve.

An infant was diagnosed as having hydrocephalus. It was determined that there was a blockage in the ventricular system of the baby's brain between the third and fourth ventricles. The blockage therefore must have involved the: Central canal Cerebral aqueduct Foramen of Luschka (lateral foramen) Foramen of Magendie (medial foramen) Interventricular foramen

cerebral aqueduct The cerebral aqueduct is the part of the ventricular system that carries cerebrospinal fluid from the third ventricle to the fourth ventricle. So, this must be the part of the ventricular system that was blocked. The central canal is the space where CSF flows through the spinal cord. It is continuous with the 4th ventricle. The foramina of Luschka (lateral aperatures) and foramen of Magendie (median aperature) are small foramina in the 4th ventricle that allow the CSF to leave the ventricular system and enter the subarachnoid space. The interventricular foramina are passages from the lateral ventricles that allow the CSF to enter the 3rd ventricle.

During surgical procedures of the neck, structures within the carotid sheath can be retracted (pulled aside) as a unit. Which of the following structures would remain in place when the carotid sheath is retracted? common carotid artery cervical sympathetic trunk internal jugular vein internal carotid artery vagus nerve

cervical sympathetic trunk The cervical sympathetic trunk lies in front of the prevertebral muscles in the prevertebral fascia. It is not part of the carotid sheath. All of the other structures listed are in the carotid sheath. The carotid sheath contains 3 vessels: internal carotid, common carotid, and internal jugular vein. It also contains 2 nerves: vagus and the superior ramus of ansa cervicalis.

A 3-year-old girl ruptured her eardrum when she inserted a pencil into her ear. Her mother took her to the emergency department after noticing that the child was crying and complaining of pain in her ear with a few drops of blood in the external auditory meatus. The attending doctor examined the child for possible injury to a nerve that runs across the eardrum. The most likely nerve to be injured is the: Auricular branch of the vagus Chorda tympani Glossopharyngeal (CN IX) Lesser petrosal Trigeminal (CN V)

chorda tympani When this girl ruptured her eardrum, she damaged the tympanic membrane. So, you need to think about the nerve and structures that are associated with the tympanic membrane (ie, on the lateral wall of the middle ear). Chorda tympani lies across the tympanic membrane, so it's possible that this nerve was injured by the pencil. The auricular branch of the vagus nerve is a small branch of the vagus that supplies afferent sensory innervation to the external acoustic meatus. This nerve is not close to the tympanic membrane The glossopharyngeal nerve and lesser petrosal nerve are associated with the promontory of the ear, which is on the medial wall of the middle ear. The trigeminal nerve is not close to the ear and would not be damaged by the injury.

At the temporomandibular joint (TMJ), hinge movements occur between the: condyle and articular eminence articular disc and articular eminence condyle and articular disc articular disc and articular cavity condyle and articular cavity

condyle and articular disc The TMJ joint is a synovial joint with two articular cavities. Each cavity is responsible for a different movement at the joint. An articular disc sits between the condylar process of the mandible on its inferior side and the mandibular fossa and articular eminence of the temporal bone on the superior side. This disc divides the joint into the two articular cavities, with one cavity acting as a hinge component and the other cavity serving as a gliding component. The lower part of the joint, between the condyle and the articular disc, is the hinge component of the joint. When the joint moves, this hinge component of the joint moves first, to initiate mandibular opening. The upper part of the joint, between the articular disc and the mandibular fossa and articular eminence of the temporal bone, creates the gliding component. During joint movement, this gliding cavity moves after the hinge component to terminate mandibular opening.

Infections may spread from the nasal cavity to the meninges along the olfactory nerves, as its fibers pass from the mucosa of the nasal cavity to the olfactory bulb via the: Cribriform plate of the ethmoid Crista galli Foramen caecum Superior orbital fissure

cribriform plate of the ethmoid. The olfactory nerve exits the skull through the cribriform plate of the ethmoid bone--an infection in the nasal cavity may be carried to the olfactory bulb by the nerves that are passing through the cribriform plate. The crista galli is a ridge on the ethmoid bone between the two sides of the cribriform plate; it provides an anchor for the falx cerebri. Foramen cecum is a small hole in the frontal bone near the anterior end of the crista galli--it transmits an emissary vein. Finally, the superior orbital fissure is a hole in the sphenoid bone that transmits many cranial nerves: the oculomotor nerve (CN III), the trochlear nerve (CN IV), the ophthalmic division of the trigeminal nerve (CN V1) and the abducens nerve (CN VI) all pass through the superior orbital fissure.

Which of the following does not conduct spermatozoa? Ampulla of the ductus deferens Duct of the seminal vesicle Epididymis Prostatic Urethra

duct of the seminal vesicle The duct of the seminal vesicle carries seminal fluid, a basic fluid containing fructose. The contents of the seminal fluid buffers the acid in the vagina and provides nutrients for sperm. The duct of the seminal vesicle joins with the ampulla of the ductus deferens (which is carying sperm) to form the ejaculatory duct. This is the first place where seminal fluid mixes with sperm. Sperm is first formed in the seminiferous tubules. They then travel from the head to the tail of the epididymis, through the ductus deferens, into the ejaculatory duct where they mix with seminal fluid, into the prostatic urethra, through the rest of the urethra, and then out the penis. So, all of the other answer choices are places that are important for the passage of sperm.

After agreeing to have no more children, a man and his wife decided he should have a vasectomy. What structure would then be surgically ligated? Ductus deferens Ejaculatory duct Epididymis Fossa navicularis Seminal vesicle

ductus deferens In a vasectomy, the ductus deferens is ligated or excised. This means that the fluid that is then ejaculated from the seminal vesicles, prostate, and bulbourethral glands has no sperm. The sperm simply degenerate in the epididymis and the proximal ductus deferens. It would not be a good idea to ligate the ejaculatory duct or the seminal vesicle because that might compromise the patient's ability to ejaculate. Just remember, another name for the ductus deferens is the vas deferens, so it make sense that the procedure to ligate this structure is called a vasectomy.

The middle nasal concha is part of what bone? Ethmoid bone Maxilla Palatine bone Sphenoid bone Vomer

ethmoid The superior and middle nasal conchae are part of the ethmoid bone. The maxillary bone is the bone that forms the midface. It forms the inferior orbital margin and contains the teeth and maxillary sinus. The palatine bone forms the posterior part of the hard palate. The sphenoid bone is an irregularly shaped bone forming the central portion of the skull. It has many parts, including a body, greater wing, lesser wing and pterygoid plates. The vomer is a thin plate of bone forming the posteroinferior part of the nasal septum.

What structure lies deepest in the parotid gland? External carotid artery External jugular vein Facial artery Facial nerve Retromandibular vein

external carotid artery The facial nerve, retromandibular vein, and external carotid artery all course through the parotid gland. From superficial to deep, they are arranged nerve, vein, artery. So, the facial nerve would be the structure most likely to be injured by a superficial injury to the parotid gland, while the external carotid artery is somewhat protected, deep in the gland. The facial artery branches from the external carotid artery before the external carotid artery enters the parotid gland. The external jugular vein is a superficial vein on the lateral surface of the neck.

Which of the following nerves is derived from the dorsal primary rami of spinal nerves? great auricular greater occipital lesser occipital phrenic supraclavicular

greater occipital The greater occipital nerve is a branch of the dorsal primary ramus of spinal nerve C2--it provides cutaneous innervation to the posterior scalp. None of the other listed nerves are dorsal primary rami. The great auricular nerve comes from the C2 and C3 branches in the cervical plexus; it provides sensory innervation to the ear and the skin below the ear. The lesser occipital nerve also comes from C2 in the cervical plexus--it innervates the skin behind the ear. The phrenic nerve is comprised of contributions from the ventral primary rami of C3, C4, and C5--it innervates the diaphragm. The supraclavicular nerves come from C3 and C4 in the cervical plexus. These cutaneous nerves provide sensory innervation to the skin of the root of the neck, upper chest, and upper shoulder.

Due to multiple salivary calculi (stones) in the submandibular duct, the submandibular gland of a 45-year-old individual was surgically removed. What major artery directly related to the gland was of special concern to the surgeon? lingual superior thyroid facial ascending pharyngeal maxillary

facial The facial artery arises from the external carotid and winds toward the inferior border of the mandible, crossing over the submandibular gland. So, if the submandibular gland was removed, the facial artery might be damaged. The lingual artery is a branch of the external carotid that runs in the floor of the mouth. It is associated with the submandibular duct, but not with the gland itself. The superior thyroid artery is a branch of the external carotid which travels anteroinferiorly to supply the upper pole of the thyroid. The ascending pharyngeal artery is a posterior branch of the external carotid which supplies blood to the pharynx. Finally, the maxillary artery is one of the two terminal branches of the external carotid artery--it supplies blood to the maxillary region, muscles of mastication, infratemporal fossa, and deep face. Take a look at Netter Plate 63 to get a better picture of this!

To study the compensatory response of selective suprahyoid muscles in elevating the hyoid bone, an experiment was designed in which the posterior belly of the digastric and stylohyoid muscles were paralyzed by drugs. The muscular branches of which of the following nerves must be chemically interrupted to produce paralysis in both muscles? inferior alveolar facial hypoglossal glossopharyngeal lingual

facial The facial nerve (CN VII) provides motor innervation to the posterior belly of the digastric and the stylohyoid muscle. The inferior alveolar nerve is a branch of the mandibular division of the trigeminal nerve (V3) that innervates mylohyoid and the anterior belly of the digastric. The hypoglossal nerve (CN XII) supplies motor innervation to the intrinsic and extrinsic muscles of the tongue, with the exception of palatoglossus. The glossopharyngeal nerve (CN IX) provides motor innervation to stylopharyngeus. Finally, the lingual nerve is a sensory branch of the mandibular division of the trigeminal nerve (V3) which supplies general sense from the anterior 2/3 of the tongue and floor of the mouth.

A sixty-four-year old man was diagnosed with an acoustic neuroma (tumor of the VIIIth cranial nerve) where it entered the temporal bone. What other cranial nerve might also be affected since this nerve uses the same foramen as the VIIIth in its course? Abducens Facial Glossopharyngeal Trigeminal Vagus

facial The facial nerve enters the temporal bone with the vestibulocochlear nerve--both cross into the internal acoustic meatus. Abducens (CN VI) crosses through the superior orbital fissure, along with the oculomotor nerve (CN III), the trochlear nerve (CN IV) and the ophthalmic division of the trigeminal nerve (CN V1). The glossopharyngeal (CN IX), vagus (CN X), and spinal accessory nerve (CN XI) all leave through the jugular foramen. Finally, the three divisions of the trigeminal nerve all leave through different foramina: V1, the ophthalmic division, exits through the superior orbital fissure; V2, the maxillary division, leaves through foramen rotundum; V3, the mandibular division, leaves through foramen ovale.

All of the following nerves exit the cranial cavity by way of bony openings located in the middle cranial fossa EXCEPT: Abducens Trochlear Oculomotor Trigeminal Facial

facial The middle cranial fossa is the part of the skull that supports the temporal lobes of the brain. It is made of the greater wings of the sphenoid and squamous parts of the temporal bones laterally and the petrous parts of the temporal bones posteriorly. See Netter Plate 6 and 7 for a better picture of this. Several cranial nerves enter foramina in the middle cranial fossa. The abducens (CN VI) crosses through the superior orbital fissure, along with the oculomotor nerve (CN III), trochlear nerve (CN IV) and ophthalmic division of the trigeminal nerve (CN V1). The trigeminal nerve (CN V) has three divisions that all leave through spaces in the middle cranial fossa. V1, the ophthalmic division, exits through the superior orbital fissure; V2, the maxillary division, leaves through foramen rotundum; V3, the mandibular division, leaves through foramen ovale. The facial nerve, however, leaves the base of the skull by passing through the internal acoustic meatus, which is in the part of the temporal bone that is in the posterior cranial fossa.

In accessing the submandibular gland in the submandibular triangle, what vessel coursing through the gland and triangle would need to be protected? External jugular vein Facial artery Maxillary artery Retromandibular vein Superior thyroid artery

facial artery The submandibular triangle is formed by the lower border of the mandible and the anterior and posterior bellies of the digastric muscle. The facial artery and facial vein course through this triangle. The facial artery lies deep to the superficial part of the submandibular gland and wraps around the mandible. The facial vein is superficial to the gland--see Netter Plate 27 for a picture. The external jugular vein is a superficial vein on the lateral side of the neck. The maxillary artery is a branch of the external carotid artery that is the main source of blood to the infratemporal fossa. The retromandibular vein is a vein that passes through the parotid gland, along with the facial artery and facial nerve. Finally, the superior thyroid artery is a branch of the external carotid that supplies blood to the superior pole of the thyroid.

A patient is unable to taste a piece of sugar placed on the anterior part of the tongue. Which cranial nerve is most likely to have a lesion? Facial nerve Glossopharyngeal nerve Hypoglossal nerve Trigeminal nerve Vagus nerve

facial nerve Taste fibers to the anterior 2/3 of the tongue come from the chorda tympani--a branch of the facial nerve. So, if the patient has lost taste sensation to the anterior 2/3 of the tongue, the patient might have a lesion of the facial nerve. The glossopharyngeal nerve (CN IX) carries taste and general sensory fibers to the posterior 1/3 of the tongue. The hypoglossal nerve (CN XII) gives motor innervation to all the muscles of the tongue (ie, all the muscles that end in glossus except for palatoglossus, which is a palate muscle innervated by the vagus). The trigeminal nerve supplies general sensory fibers to the anterior 2/3 of the tongue via the lingual nerve, a branch of V3. Remember, the chorda tympani fibers jump on the lingual nerve to reach the tongue, but the chorda tympani fibers originate from the facial nerve! Finally, the vagus nerve supplies general and taste sensation to a very small part of the posterior tongue, right next to the epiglottis.

A deep laceration of the face in the middle of the parotid gland could affect the: External jugular vein Facial nerve Glossopharyngeal nerve Hypoglossal nerve Lingual artery

facial nerve The facial nerve travels through the parotid gland--it could become injured if there was a deep laceration through the parotid gland. Two other structures found within the parotid gland which might be damaged: the retromandibular vein and the external carotid artery. The nerve is the most superficial structure in the gland. Then, the vein is under the nerve, and the artery is the deepest structure in the gland. The external jugular vein is a superficial structure on the lateral neck, so it's not really close to the parotid gland. The glossopharyngeal nerve is closely related to the stylopharyngeus muscle--it sweeps along the back of this muscle. It is not related to the parotid gland. The hypoglossal nerve travels laterally to the carotid vessels and then enters the floor of the mouth. This means that it travels inferior to the region of the parotid gland. Finally, the lingual artery is found in the floor of the mouth--far from the parotid gland!

The inferior sagittal sinus is found in the free edge of what structure? Diaphragma sellae Falx cerebelli Falx cerebri Filum terminale Tentorium cerebelli

falx cerebri The falx cerebri is a crescent-shaped, sagittally-oriented fold of dura mater lying between cerebral hemispheres. The inferior sagittal sinus runs in the inferior margin of the falx cerebri. The falx cerebelli is a small fold of dura mater lying between cerebellar hemispheres--it is the location of the occipital sinus. The diaphragma sellae is a piece of dura mater which forms the roof of the hypophyseal fossa; it is pierced by the stalk of the hypophysis. The tentorium cerebelli is a tent-like sheet of dura mater covering of cerebellum, oriented somewhat transversely. The straight sinus is found in the junction of the falx cerebri and tentorium cerebelli. The filum terminale is an extension of the pia mater below the end of the spinal cord at L2. It is a structure of the spinal cord, not the brain.

Which of the following developmental processes is least likely to be involved in the differentiation of male external genitalia from the indifferent state? Descent of the gonads into the labio-scrotal folds Fusion of the urogenital folds Elongation of the phallus Formation of new erectile bodies

formation of new erectile bodies Males and females have analogous erectile bodies. The three main erectile bodies in males are 2 corpus cavernosa and a corpus spongiosum. Females have 2 corpus cavernosa plus 2 vestibular bulbs which are analogs to the corpus spongiosum. So, females actually have more erectile bodies than males. Development differs, however, in many other ways. Remember: the gubernaculum pulls the testes to descend into the scrotum, but the ovaries stop their descent and remain in the pelvis. The urogenital folds fuse in males to create the raphe of the penis. However, they stay open in females to create the labial minora and perineal raphe. Finally, the phallus elongates in males but not in females.

The cell bodies of the taste fibers from the anterior two-thirds of the tongue are located in the: Geniculate ganglion Otic ganglion Pterygopalatine ganglion Submandibular ganglion Trigeminal ganglion

geniculate ganglion The geniculate ganglion is the sensory ganglion of the facial nerve--it holds the cell bodies of the neurons that carry taste sensations from the anterior 2/3 of the tongue. These fibers then travel on the chorda tympani, which carries the fibers to the lingual nerve. The chorda tympani fibers then jump on the lingual nerve so they can reach the tongue and provide taste sensation to the anterior 2/3. The otic ganglion is the ganglion where parasympathetic fibers synapse before innervating the parotid gland. The lesser petrosal nerve is a branch of the glossopharyngeal nerve (CN IX) that carries presynaptic parasympathetic fibers to the otic ganglion. These fibers synapse in the otic ganglion, and the postsynaptic fibers travel on the auriculotemporal nerve to the parotid gland.

When one presses the tongue tip against the anterior (incisor) teeth, which of the following muscles must contract? Styloglossus Hyoglossus Genioglossus Superior longitudinal Verticalis

genioglossus Genioglossus is a large, fan shaped muscle with many actions: its inferior fibers protrude the tongue, its middle fibers depress the tongue, and its superior fibers draw the tip back and down. So, since the tongue is being protruded, the inferior fibers of genioglossus must be contracting. Styloglossus retracts and elevates the tongue. Hyoglossus depresses the sides of tongue and retracts the tongue. The superior longitudinal and verticalis muscles are intrinsic muscles of the tongue. They help shape the tongue for speech and chewing. All of these muscles are innervated by the hypoglossal nerve!

Which nerve provides cutaneous innervation to the skin of the angle of the mandible? Auriculotemporal nerve Lesser petrosal nerve Buccal branches of VII Marginal mandibular nerve Great auricular nerve

great auricular nerve The great auricular nerve is a branch of the cervical plexus that provides cutaneous innervation to the skin of the ear and skin below the ear, including the angle of the mandible. The auriculotemporal nerve is a branch of the mandibular division of the trigeminal nerve (V3) with two important functions. First, it carries postganglionic parasympathetic fibers to the parotid gland. Second, the auriculotemporal nerve provides sensory innervation to the skin of anterosuperior ear, part of the external auditory meatus, and the temporomandibular joint. The lesser petrosal nerve is not a sensory nerve--it is a branch of the glossopharyngeal nerve that carries preganglionic fibers to the otic ganglia. Finally, the buccal branches and marginal mandibular branches of the facial nerve are motor nerves only--not sensory nerves! The buccal branches of the facial nerve innervate the buccinator and the other muscles of facial expression above the lip. The marginal mandibular branch innervates the muscles of facial expression of the lower lip and chin.

Which of the following is a dorsal ramus of a spinal nerve? Supraclavicular n. Transverse cervical n. Great auricular n. Greater occipital n. Lesser occipital n.

greater occipital nerve The greater occipital nerve is the cutaneous branch of the dorsal primary ramus of spinal nerve C2 - it provides cutaneous innervation to the posterior scalp. The other nerves listed are all nerves from the cervical plexus, which is formed by ventral primary rami C1-C4. The supraclavicular nerves come from C3 and C4 in the cervical plexus. These cutaneous nerves provide sensory innervation to the skin of the root of the neck, upper chest, and upper shoulder. The great auricular nerve comes from C2 and C3 branches; it provides sensory innervation to the ear and the skin below the ear. The lesser occipital nerve comes from C2 only - it innervates the skin behind the ear. The transverse cervical nerve is also a cutaneous branch from the cervical plexus--from C2 and C3, providing sensory innervation to the skin of the neck anteriorly.

A patient who has sustained a fracture to the middle cranial fossa following a fall from a height, might have any of these nerves injured EXCEPT: Trigeminal Oculomotor Abducens Trochlear Hypoglossal

hypoglossal The middle cranial fossa is the part of the skull that supports the temporal lobes of the brain. It is made of the greater wings of the sphenoid and squamous part of the temporal bones laterally and the petrous part of the temporal bones posteriorly. Several cranial nerves enter foramina in the middle cranial fossa; all of these nerves might have been damaged in the fall. The trigeminal nerve (CN V) has three divisions that all leave through spaces in the middle cranial fossa. V1, the ophthalmic division, exits through the superior orbital fissure; V2, the maxillary division, leaves through foramen rotundum; V3, the mandibular division, leaves through foramen ovale. The oculomotor nerve (CN III) crosses through the superior orbital fissure, along with abducens (CN VI), the trochlear nerve (CN IV) and the ophthalmic division of the trigeminal nerve (CN V1). So, all of these nerves might have been damaged in the fall. The hypoglossal nerve, however, leaves the base of the skull by passing through the hypoglossal canal, which is in the occipital bone and the posterior cranial fossa. It is not likely that this nerve was injured in the fall.

An elderly man presented with severe pain beneath the left eye, radiating into the lower eyelid, lateral side of the nose and upper lip. What nerve was involved? Buccal Infraorbital Mental Supratrochlear Zygomatic

infraorbital The infraorbital nerve is a cutaneous nerve from the maxillary division of trigeminal nerve (V2). It innervates the skin of the lateral nose, lower eyelid, upper lip and zygomatic region. This is exactly where this man's pain is, so it seems like his pain must be transmitted on the infraorbital nerve. The buccal branch of the trigeminal nerve is part of the mandibular division (V3)--this nerve provides sensory innervation to the skin of the cheek and the mucosal lining the cheek. It is not a motor nerve--only sensory! (NOTE: The buccal nerve is NOT the same as the buccal branch of the facial nerve. The buccal branch of the facial nerve is a motor nerve only--it innervates several muscles of facial expression. It does not have a sensory component--only motor!) The mental nerve is a branch of the inferior alveolar nerve, which is a branch of the mandibular division of the trigeminal nerve (V3). It provides sensory innervation to the skin of the chin and lower lip. The supratrochlear nerve is a branch of the frontal nerve, from the ophthalmic division of the trigeminal nerve. It gives sensory innervation to the skin of the medial forehead and the medial part of the upper eyelid. The zygomatic nerve is part of the maxillary division of the trigeminal nerve (V2). It provides sensory innervation to the skin of the face lateral and superior to the orbit.

A structure which is homologous to the male scrotum: Labia minora Labia majora Glans Shaft of corpus cavernosum

labia majora The labia majora and scrotum are homologous structures. The labia minora is the female counterpart of the pentscrotal raphe. The glans of the clitoris and glans of the penis are homologous structures. Finally, the shaft of the corpus cavernosum in the female is the shaft of the clitoris, which is homologous to the shaft of the penis.

An infection in which scalp layer is likely to spread most readily? Skin Connective tissue layer Aponeurotic layer Loose areolar tissue Pericranium

loose areolar tissue Loose areolar tissue is another name for the loose connective tissue layer of the scalp. Pus or blood can spread easily in this layer, and infections in this layer can pass into the cranial cavity through emissary veins. So, infections in the loose connective tissue can pass into intracranial structures such as the brain and meninges. This can also be called the "danger layer" of the scalp. Remember--the scalp is comprised of the following layers, from superficial to deep: Skin, Connective tissue, Aponeurosis, Loose connective tissue, and Pericranium. (SCALP!) Although layer 2 is a connective tissue layer, too, this layer is a bit thicker and is not a place where infections can easily spread.

The "danger zone" of the scalp is recognized as which of the following layers? Galea aponeurotica Loose connective tissue Pericranium Skin Subcutaneous connective tissue

loose connective tissue The scalp is comprised of the following layers, from superficial to deep: Skin, Connective tissue, Aponeurosis, Loose connective tissue, and Pericranium. If you take the first letter of each, it spells SCALP. So, now that you know the order of the layers, you need to figure out which one is the danger zone--the place where infections can spread very quickly. And that layer is layer 4, the loose connective tissue. Pus or blood can spread easily in this layer, and infections in this layer can pass into the cranial cavity through emissary veins. So, infections in the loose connective tissue can pass into intracranial structures such as the brain and meninges. Although layer 2 is a connective tissue layer, too, this layer is a bit thicker and is not a place where infections can easily spread.

As a result of a face lift operation, a 46-year-old woman noticed an asymmetry of the inferior lip and could not fully depress the angle of her mouth on the right side. Which of the following nerves was most likely damaged during the surgery? zygomatic (VII) buccal (VII) mental (V3) marginal mandibular (VII) infraorbital (V2)

marginal mandibular Depressor anguli oris is the muscle that depresses the angle of the lip--it is innervated by the marginal mandibular branch of the facial nerve. So, if the marginal mandibular branch of the facial nerve was injured, depressor anguli oris would be paralyzed. The zygomatic branches of the facial nerve innervate the muscles of facial expression that are right around the eye, including orbicularis oculi. The buccal branches of the facial nerve innervate the buccinator muscle and other muscles of facial expression that are near the upper lip, like levator anguli oris and levator labii superioris. The mental and infraorbital branches of the trigeminal nerve provide sensory innervation to the skin of the face--they do not innervate any muscles! The mental nerve, a branch of V3 (mandibular division), innervates the skin of the chin and the lower lip. The infraorbital nerve, a branch of V2 (maxillary division), innervates the skin of the lateral nose, lower eyelid, upper lip, and zygomatic region.

An 84-year old woman suffers a stroke, with paralysis on the right side of her body. Neurological tests show that the intracerebral hemorrhage has interrupted the blood supply to the posterior part of the frontal, the parietal and medial portions of the temporal lobes of the left cerebral hemisphere. Which vessel was involved? Anterior cerebral artery Great cerebral vein Middle cerebral artery Middle meningeal artery Posterior cerebral artery

middle cerebral artery The middle cerebral artery supplies blood to most of the lateral surface of cerebral hemispheres, and the temporal pole, including the frontal, parietal, and medial portions of the temporal lobes. So, the specific damage to the temporal lobe suggests that the middle cerebral artery was disrupted. The other arteries listed do not distribute to the same territory. The anterior cerebral artery supplies the medial and superior surfaces of the brain, including the frontal pole. The posterior cerebral artery supplies the inferior surface of the brain and the occipital pole. Strokes occur in arteries, not veins, so that's one reason why the great cerebral vein is not correct. Veins also drain regions of blood--they don't supply blood to areas. The middle meningeal artery supplies blood to the dura mater and the cranial vault bones--it does not supply blood to the brain.

Most paranasal sinuses and/or air cells drain, directly or indirectly, into the: Inferior meatus Middle meatus Superior meatus Nasal vestibule Sphenoethmoidal recess

middle meatus The middle meatus contains the semilunar hiatus, which receives drainage from the frontonasal duct (draining the frontal sinus), the anterior ethmoidal air cells, and the maxillary sinus. So, it's draining most of the paranasal sinuses. The inferior meatus receives the nasolacrimal duct which conducts tears from the orbit. The superior meatus receives drainage from the posterior ethmoidal air cells. The nasal vestibule is the opening of the nose--none of the sinuses drain directly into this area. The sphenoethmoidal recess is the opening for the sphenoethmoidal sinus.

Irrigation of the maxillary sinus through its opening is a supportive measure to accelerate the resolution of a maxillary sinus infection. Which of the following nasal spaces is the most likely approach to the sinus opening? choana inferior meatus middle meatus sphenoethmoidal recess superior meatus

middle meatus The middle meatus contains the semilunar hiatus, which receives drainage from the maxillary sinus, the frontonasal duct (draining the frontal sinus), and the anterior ethmoidal air cells. The maxillary sinus is draining into the middle meatus, so it would be easiest to approach the maxillary sinus through this space. The choana is the space where the nasal cavity opens into the nasopharynx--it is found at the very posterior border of the nasal cavity. The inferior meatus receives the nasolacrimal duct which conducts tears from the orbit. The sphenoethmoidal recess is the opening for the sphenoethmoidal sinus. Finally, the superior meatus receives drainage from the posterior ethmoidal air cells.

The most likely source of blood in a patient with an epidural hemorrhage is: Vertebral artery Middle meningeal artery Superior cerebral veins Anterior cerebral artery Circle of Willis

middle meningeal artery The middle meningeal artery supplies most of the dura mater and the bones of the cranial vault. It is this artery or vein that is usually ruptured in an epidural hemorrhage. The vertebral artery carries blood to the deep neck, cervical spinal cord, and hindbrain. It does not supply blood to the dura. Superior cerebral veins drain blood into the superior sagittal sinus. When injured, they bleed into the subdural space and cause a subdural hematoma. The anterior cerebral artery supplies blood to the frontal pole of the brain. It is not found near the epidural space. Finally, the circle of Willis is an important anastomosis at the base of the brain between the following arteries: posterior cerebral arteries, posterior communicating arteries, internal carotid arteries, anterior cerebral arteries, and anterior communicating arteries.

An intrahepatic blockage of the portal venous outflow may cause intestinal blood to drain via portal-systemic anastomoses into the: Superior gluteal vein Middle rectal vein Splenic vein Renal vein Inferior phrenic vein

middle rectal vein There are four portal-caval anastomoses in the body. First, between the superior rectal veins in the portal system and the middle and inferior rectal veins in the caval system. Second, between the esophageal veins that go to the left gastric vein (portal) and the esophageal veins that go to the azygos system (caval). Third, between the paraumbilical veins of the portal system and the veins of the anterior abdominal wall that drain into the inferior vena cava. Fourth, between the colic veins of the portal system and the retroperitoneal veins of the caval system. So middle rectal is the right answer. The superior gluteal vein, renal vein, and inferior phrenic vein are all part of the caval system; the splenic vein is part of the portal system.

A surgeon doing a deep cervical lymph node dissection may encounter these structures upon the anterior surface of the anterior scalene muscle EXCEPT: Phrenic nerve Subclavian vein Middle trunk of the brachial plexus Transverse cervical artery Cervical part of the thoracic duct

middle trunk of the brachial plexus The roots of the brachial plexus are between the anterior and middle scalene muscles; the trunks of the brachial plexus which come from these roots travel in the posterior triangle. So, the trunks of the brachial plexus are not related to the anterior surface of the anterior scalene muscle. The phrenic nerve is comprised of contributions from the ventral primary rami of C3, C4, and C5. This nerve courses along the anterior surface of the anterior scalene. The brachiocephalic vein also crosses the anterior surface of the anterior scalene as it reaches its connection with the internal jugular vein. The transverse cervical artery is a branch of the thyrocervical trunk which crosses the anterior scalene. Finally, the cervical part of the thoracic duct dumps into the brachiocephalic vein on the left side of the neck. So, on the left side of the neck, the thoracic duct lies on the anterior part of the anterior scalene.

A 47-year-old woman is diagnosed as having a thyroid tumor. Surgery to remove the cancerous growth is undertaken. In which triangle of the neck will the surgeon make an incision to gain access to the gland? Carotid Muscular Subclavian Submandibular Submental

muscular The muscular triangle is found in the anterior triangle of the neck. It is bounded by the midline as its medial border, the superior belly of the omohyoid as its superolateral border, and sternocleidomastoid as its inferolateral border. The strap muscles, thyroid, and parathyroids are found in this triangle. Since the thyroid is found at the midline, deep to the strap muscles, you would need to cut into this triangle to perform surgery on the thyroid gland.

The muscle which separates the submandibular triangle from the paralingual space is the: Digastric, posterior belly Hyoglossus Mylohyoid Stylohyoid Styloglossus

mylohyoid The submandibular triangle is a space bounded by the anterior and posterior bellies of the digastric and the body of the mandible. It is found superficial to the mylohyoid muscle. The submandibular triangle contains the superficial submandibular gland, stylohyoid muscle, facial artery and facial vein. The paralingual space is a space found deep to the mylohyoid muscle, which makes the mylohyoid the dividing line between these two distinct spaces. It is bounded by the lateral tongue (including the hyoglossus, styloglossus, and genioglossus muscles), hyoid bone, and oral mucosa. It contains the deep portion of the submandibular gland, the submandibular duct and the sublingual caruncle, lingual nerve and submandibular ganglion, hypoglossal nerve, and sublingual gland and fold.

Which of the following hyoid muscles is an important landmark in both the anterior and posterior triangles of the neck? geniohyoid mylohyoid omohyoid sternohyoid stylohyoid

omohyoid The omohyoid muscle is an important landmark in both the anterior and posterior triangles of the neck. In the anterior triangle, the superior belly of the omohyoid muscle serves as the superolateral border of the muscular triangle and the anterior border of the carotid triangle. In the posterior triangle, the inferior belly of the omohyoid muscle divides the omoclavicular triangle from the occipital triangle. So, in both the anterior and posterior triangles, the omohyoid is an important muscle that subdivides the triangles.

An elderly woman complained of a severe pain, felt above the right eye radiating to the upper eyelid, side of the nose and forehead. Branches of which of the following nerves convey pain sensations from areas of the skin described? maxillary (V2) greater auricular nerve ophthalmic (V1) mandibular (V3) facial (VII)

ophthalmic (V1) The ophthalmic division of the trigeminal nerve provides sensory innervation to the skin of the nose, upper eyelid, and forehead. This is exactly where this woman feels pain, so the ophthalmic division of the trigeminal nerve must be the nerve transmitting the pain. The maxillary division of the trigeminal nerve (V2) provides sensory innervation to the skin of the side of the nose, the cheek, lower eyelid, and upper lip. The mandibular division of the trigeminal nerve (V3) provides sensory innervation to the skin of the chin, lower lip, and lower jaw. The great auricular nerve is a branch of the cervical plexus which supplies sensory innervation to the ear region. Finally, the facial nerve is mostly a motor nerve--it only supplies taste to the anterior 2/3 of the tongue and gives some sensory innervation to the skin of the exernal auditory meatus.

The teeth and gums separate the oral cavity proper from the: Nasal cavity Oral vestibule Oropharynx Paralingual space Submandibular space

oral vestibule

A patient is unable to wink, what muscle is affected? frontalis levator palpebrae superioris orbicularis oculi superior tarsal zygomaticus major

orbicularis oculi Orbicularis oculi is a muscle of facial expression that closes the eyelid for winking. It is innervated by the temporal and zygomatic branches of the facial nerve. Frontalis is the anterior belly of the epicranius muscle; it elevates the eyebrows and wrinkles the forehead. It is innervated by the temporal branches of the facial nerve (VII). Levator palpebrae superioris elevates the upper eyelid; it is innervated by the oculomotor nerve (III). The superior tarsal muscle is another muscle that elevates the eyelid, but it lifts the eyelid invoulntarily. It is innervated by the cervical sympathetic trunk; checking whether this muscle is functioning is a good test to see whether the cervical sympathetic trunk is intact. Finally, zygomaticus major is a muscle at corner of the mouth which elevates and draws the corner of the mouth laterally. It is innervated by the zygomatic and buccal branches of the facial nerve.

You are examining a patient who has a pituitary tumor involving the cavernous sinus. While doing a preliminary eye exam, you suspect the right abducens nerve of the patient has been damaged by the tumor. In which direction would you have the patient turn his right eye to confirm the defect? Inward Outward Downward Down and out Down and in Upward Up and out Up and in

outward To understand this question, you need to understand how the motions of the eye are tested. Since the actions of the extraocular muscles are complex, it is necessary to turn the eye to a position where a single action of each muscle predominates when evaluating the individual muscles. For the superior and inferior recti, turning the eye outward (abduction) by approximately 25 degrees places the superior rectus in position to raise the eye and the inferior rectus to lower the eye. Similarly, turning the eye inward (adduction) approximately 50 degrees places the inferior oblique in position to raise the eye and the superior oblique to lower the eye. The medial and lateral recti may be checked while the eye is staring straight ahead since they have simple planar actions. In this case, you're interested in testing an "easy" muscle. Since the lesion appears to be in the abducens, which innervates the lateral rectus muscle, you could just ask the patient to turn the eye outward. If the patient could not do this, it would confirm that there was a lesion in the abducens nerve, since the muscle responsible for lateral movement of the eye would be paralyzed. Also remember--a tumor in the cavernous sinus could affect many nerves. The oculomotor nerve (CN III), trochlear (CN IV), ophthalmic division of trigeminal (CN V1), and abducens (CN VI) all pass through the cavernous sinus.

The inner lining of the eyelid is called the: Orbital septum Palpebral conjunctiva Periorbita Sclera Tarsal plate

palpebral conjunctiva The palpebral conjunctiva is the thin membrane that lines the eyelid. It is continuous with the bulbar conjunctiva which lines the eyeball. The orbital septum is a weak membrane that spans from the tarsal plates to the margins of the orbit where it becomes continuous with the periosteum. It contains orbital fat and can limit the spread of infection in the orbit. The periorbita is the periosteum lining covering the bones forming the orbit. The sclera is the outer fibrous layer of the eyeball. Finally, the tarsal plate is a thin, cardboard-like layer of connective tissue in the eyelids which forms the "skeleton" of the eyelids.

Damage to the facial nerve near the stylomastoid foramen would likely cause each of the following motor deficits EXCEPT: Paralysis of the buccinator muscle Inability to whistle Paralysis of the muscles that elevate the mandible Inability to close the lips

paralysis of the muscles which elevate the mandible The masseter, medial pterygoid, and temporalis are all muscles that elevate the mandible. These muscles are innervated by branches of V3, the mandibular division of the trigeminal nerve. So, none of these muscles would be denervated if the facial nerve was injured. The other actions discussed all involve the muscles of facial expression, which are innervated by the facial nerve (CN VII). The buccinator muscle is innervated by the buccal branches of the facial nerve--this muscle pulls the corner of mouth laterally and presses the cheek against the teeth. The orbicularis oris is the muscle that allows for whistling or closing the lips; this muscle is innervated by the buccal branches of the facial nerve.

The vestibular bulbs/bulb of the corpus spongiosum are firmly attached to the: Perineal membrane Superior pubic rami Ischiopubic rami Pubic symphysis Ischial tuberosities

perineal membrane The bulbs of the vestibule/bulb of the corpus spongiosum are pieces of erectile tissue that attach to the perineal membrane. They are covered by the bulbospongiosis muscle. The ischiopubic rami, pubic symphysis, and ischial tuberosities are bony structures important for defining the boundaries of the perineum. The crura of the corpora cavernosa attach to the ischiopubic rami and the perineal membrane.

The chorda tympani enters the infratemporal fossa after it exits the: Stylomastoid foramen Foramen spinosum Foramen lacerum Petrotympanic fissure

petrotympanic fissure The chorda tympani exits the skull through the petrotympanic fissure. The stylomastoid foramen is the space that the facial nerve travels through to leave the skull--remember, the chorda tympani has already separated away from the facial nerve by this point. Foramen spinosum is a hole in the base of the skull that transmits the middle meningeal artery and vein. Finally, foramen lacerum is a ragged foramen that is an artifact of a dried skull. In life, it is closed by cartilage. Nothing passes directly through foramen lacerum, although both greater and deep petrosal nerve pass through some of the cartilage that fills it, in order to enter the pterygoid canal at the anterior margin of foramen lacerum.

What part of the ischioanal (ischiorectal) fossa extends deep to the sacrotuberal ligament? Anterior recess Genital hiatus Posterior recess Pudendal canal

posterior recess The ischioanal fossa is a space found on both sides of the anal canal. It is bounded laterally by the obturator internus, superiorly by the pelvic diaphragm, and medially by the pelvic diaphragm and anus. It is the area that is lateral to the anal canal and inferior to the pelvic diaphragm. The anterior recesses are the parts of the ischioanal fossa that extend above the perineal membrane, and the posterior recesses extend deep to the sacrotuberal membrane and superior to the gluteus maximus. The genital hiatus is the place where the pelvic diaphragm splits to allow the urethra/vagina and anus to pass through. The pudendal canal travels from the lesser sciatic foramen, where its contents enter the perineum. It contains the internal pudendal artery, internal pudendal vein, and pudendal nerve.

Which structure is found only in males? Anterior recess of ischoianal fossa Genital Hiatus Ischiocavernosus muscle Rectovesical pouch Sphincter urethrae muscle

rectovesicular pouch The rectovesicular pouch is a reflection of the peritoneum between the rectum and the bladder. It can only be found in males because females have the uterus sitting between the rectum and the bladder. This means that females have two pouches created by reflections of peritoneum draped over the pelvic viscera: the rectouterine and vesicouterine pouches. The ischioanal fossa is the fat filled space located lateral to the anal canal and inferior to the pelvic diaphragm. It has an anterior recess that extends superior to the perineal membrane and sphincter urethrae muscle, and it is found in both males and females. The genital hiatus is also found in both sexes--it is the place where the urethra/vagina and anus exit the pelvic diaphragm. The ischiocavernosis muscle compresses the corpus cavernosum of the penis or clitoris. Finally, the sphincter urethrae encircles and compresses the urethra in both sexes.

A 35 year old woman was diagnosed with an adenoma of the thyroid gland. This required excision of the lower pole (left lobe) of the gland and ligation of the artery supplying that region. Which of the following nerves accompanying the artery is most likely to be damaged if the surgeon is not careful? External branch of the superior laryngeal Internal branch of the superior laryngeal Recurrent laryngeal Vagus proper Sympathetic trunk

recurrent laryngeal The inferior thyroid artery supplies the lower lobes of the thyroid. It is a branch of the thyrocervical trunk. Branches of this artery cross the recurrent laryngeal nerve, so this is the nerve that might be damaged during the surgery. The external branch of the superior laryngeal nerve runs with the superior thyroid artery--this artery and nerve might be damaged when removing the superior lobes of the thyroid. The internal branch of the superior laryngeal nerve runs with the superior laryngeal artery, piercing the thyrohyoid membrane. The vagus nerve proper runs with the common carotid artery, in the carotid sheath. It gives off the recurrent laryngeal nerve, but it does not come as close to the inferior thyroid artery. Finally, the sympathetic trunk is in the deep neck, near the vertebral bodies. It should not be damaged in thyroid surgery.

Which muscle is innervated by a branch of the ansa cervicalis? Sternocleidomastoid Platysma Sternohyoid Trapezius

sternohyoid The sternohyoid muscle is one of the strap muscles which runs from the manubrium and the sternal end of the clavicle to the hyoid bone. It is innervated by the ansa cervicalis, and it depresses and stabilizes the hyoid bone. Platysma is a muscle of facial expression; it is innervated by the cervical branch of CN VII, the facial nerve. Sternocleidomastoid and trapezius are both innervated by the accessory nerve (CN XI). Sternocleidomastoid receives fibers from C2 and C3 for proprioception, while trapezius receives proprioceptive fibers from C3 and C4.

In performing a thyroidectomy, caution should be exercised when ligating (tying) the inferior thyroid artery, as it lies in a very close relationship to which nerve? ansa cervicalis hypoglossal phrenic recurrent laryngeal vagus

recurrent laryngeal The recurrent laryngeal nerve crosses the inferior thyroid artery near the lower lobe of the thyroid. This means that the recurrent laryngeal nerve would be at risk in any surgery involving the inferior thyroid artery or the inferior poles of the thyroid. The recurrent laryngeal nerve becomes the inferior laryngeal nerve at the inferior border of cricopharyngeus, and this nerve continues on to innervate all the muscles of the larynx with the exception of cricothyroid. So, you really need to take care to protect the recurrent laryngeal nerve--injuring this structure could lead to hoarseness, permanent loss of voice, or even death due to a laryngeal spasm. None of the other listed nerves are related to the inferior thyroid artery. Ansa cervicalis is a branch of the cervical plexus which hangs in front of the internal jugular vein. It innervates the strap muscles. The hypoglossal nerve winds behind the internal jugular vein, then sweeps around lateral to the carotid vessels and into the floor of the mouth, where it innervates the tongue muscles. The phrenic nerve lies on top of the anterior scalene muscles--it then travels through the thorax to innervate the diaphragm. Finally, the vagus is found in the carotid sheath--it is associated with the common carotid and the internal carotid arteries.

A man has metastatic carcinoma and enlarged deep cervical lymph nodes. One of his symptoms is a hoarse voice, barely heard above a whisper. Subsequently he succumbs to the disease and at autopsy it is found that one of the enlarged nodes has put pressure on a nerve in the tracheoesophageal groove. Presumably this was the reason for the hoarse voice. What nerve was involved? External branch of the superior laryngeal nerve Internal branch of the superior laryngeal nerve Nerve to the cricothyroid muscle Pharyngeal branch of the glossopharyngeal nerve Recurrent laryngeal branch of vagus

recurrent laryngeal branch of the vagus Damage to the recurrent laryngeal nerve is one possible cause of hoarseness. The recurrent laryngeal nerve changes its name to the inferior laryngeal nerve at the level of the inferior border of the cricoid cartilage. The inferior laryngeal nerve goes on to innervate all the intrinsic muscles of the larynx except cricothyroideus. So, if this nerve innervating all the muscles of the larynx was damaged, a patient would have a hoarse voice. A very significant piece of information that tells you that the recurrent laryngeal nerve was injured is the location of the injury-- remember, recurrent laryngeal travels in the tracheoesophageal g

A 6 mo. old male was brought to the pediatric clinic by his parents because of leakage of urine from the ventral surface of his penis. This congenital condition, hypospadias, is due to incomplete ventral closure of a component of the penis. Which of the below structures would be partially open for urine to take such a course? Shaft of corpus cavernosum Membranous urethra Glans Shaft of corpus spongiosum

shaft of the corpus spongiosum Since the urine is leaking through the ventral side of the penis, it must be leaking through a defect in the spongy urethra. The spongy urethra is contained in the corpus spongiosum, so it follows that the corpus spongiosum must be open. The membranous urethra is a brief portion of the urethra extending from the bottom of the prostate to the top of the corpus spongiosum. A defect here would not cause leakage on the ventral surface of the penis. The corpora cavernosa are erectile bodies that lie beside the corpus spongiosum. They are not involved with the flow of urine or the urethra. The glans of the penis is at the tip - if this structure failed to close, there would be abnormal leakage from the tip of the penis, not the ventral surface.

The perineum is bounded by all of the following skeletal elements except: coccyx ischiopubic ramus spine of ischium symphysis pubis

spine of the ischium The bounderies of the perineum are as follows. Anterior: pubic symphysis; Anterolateral: ischiopubic rami; Lateral: ischial tuberosities; Posterolateral: sacrotuberous ligament; Posterior: tip of the coccyx. The spine of the ischium, which projects into the pelvis toward the lateral pelvic wall, does not make up a boundary of the perineum

One of the motor components of the cervical plexus, the ansa cervicalis, innervates all of the following muscles except the: Omohyoid Sternohyoid Sternothyroid Stylohyoid Thyrohyoid

stylohyoid The stylohyoid muscle runs from the posterior portion of the digastric muscle to the body of the hyoid bone. It elevates and retracts the hyoid, and it is innervated by the facial nerve (CN VII). Omohyoid, sternohyoid, sternothyroid, and thyrohyoid are all strap muscles which are innervated by ansa cervicalis (although thyrohyoid receives its branch via the hypoglossal nerve).

A stab wound just above the left clavicle, lateral to the sternocleidomastoid muscle, may be life-threatening because of the possibility of injury to the: Brachial plexus Internal jugular vein Axillary artery Subclavian artery Thoracic duct

subclavian artery. The subclavian artery crosses between the anterior and middle scalene muscles, which means that it is also posterior to the sternocleidomastoid muscle. Lateral to the sternocleidomastoid muscle, the subclavian artery runs just above the clavicle. Eventually, this artery descends toward the upper limb. So, it is in the right position to be injured in the stabbing. The question also mentions that the wound was life-threatening, so you should really be looking for an answer about an artery. The brachial plexus is also in the area of the stab wound, but a brachial plexus injury would not be life-threatening. The internal jugular vein is located medial and posterior to the sternocleidomastoid muscle--it is not likely that this vein would be cut. The axillary artery is a continuation of the subclavian artery beneath the level of the first rib, which is inferior to the area of interest. The thoracic duct, which dumps into the brachiocephalic vein on the left side, might be injured in the stabbing, but this would not be a life-threatening problem.

A person receives a shallow knife wound just behind the sternocleidomastoid muscle and about 1 1/2 inches above the clavicle. There is an immediate numbness of the skin below the wound and over the acromion and clavicle. The nerve most likely severed was the greater auricular lesser occipital supraclavicular suprascapular transverse cervical

supraclavicular The supraclavicular nerves come from C3 and C4 in the cervical plexus. These cutaneous nerves provide sensory innervation to the skin of the root of the neck, upper chest, and upper shoulder. These nerves innervate the skin right above the clavicle, so they are the correct answer. The great auricular nerve comes from C2 and C3 branches in the cervical plexus; it provides sensory innervation to the ear and the skin below the ear. The lesser occipital nerve comes from C2 in the cervical plexus - it innervates the skin behind the ear. The transverse cervical nerve is also a cutaneous branch from the cervical plexus - it is from C2 and C3 and provides sensory innervation to the skin of the neck anteriorly. The suprascapular nerve is not a cutaneous nerve - it comes from the superior trunk of the brachial plexus and provides motor innervation to supraspinatus and infraspinatus.

Which of the following structures is located in the vestibule of the oral cavity? Tongue Opening of the parotid duct Opening of the submandibular duct Sublingual fold Uvula

the opening of the parotid duct The oral vestibule is the space in the mouth lying between the lips and the teeth. See Netter Plate 45 for an illustration of the mouth and vestibule--although the vestibule isn't labeled, you can get some idea of where it lies. You can also see that the parotid duct is opening into this space . The parotid duct drains the parotid gland; it crosses the masseter to enter the cheek and drain into the oral cavity. The parotid duct drains into the cheek near the upper 2nd molar tooth.

Pain elicited from an infected facial wound is primarily conveyed by what nerve? Facial Great auricular Hypoglossal Transverse cervical Trigeminal

trigeminal The trigeminal nerve is the nerve that supplies sensory innervation to the skin of the face, so pain sensations will be carried through this nerve. It has 3 divisions. The ophthalmic division (V1) is a sensory nerve that passes through the superior orbital fissure and supplies sensory innervation to the eyeball, conjunctiva, nasal mucosa, medial portion of the nose, upper eyelid, forehead, and scalp. The maxillary division (V2) is a sensory nerve that provides sensory innervation to the cheek, upper lip, lateral portion of nose and lower eyelid. The mandibular division (V3) is a sensory and motor nerve--it supplies the lower lip and chin and the lateral portion of the cheek. V3 also provides motor innervation to the muscles of mastication, tensor veli palatini, mylohyoid, the anterior belly of the digastric, and tensor tympani. The facial nerve does not provide sensory innervation to the skin of the face--it provides motor innervation to the muscles of facial expression. The great auricular nerve and transverse cervical nerve are branches of the cervical plexus. The great auricular nerve supplies the skin of the ear and the skin below the ear, while the transverse cervical nerve supplies the skin of the anterior neck. The hypoglossal nerve (CN XII) provides motor innervation to the muscles of the tongue.

You are testing the extraocular muscles and their innervation in a patient who periodically experiences double vision. When you ask him to turn his right eye inward toward his nose and look downward he is able to look inward, but not down. Which nerve is most likely involved? Abducens Nasociliary Oculomotor, inferior division Oculomotor, superior division Trochlear

trochlear Since the actions of the extraocular muscles are complex, it is necessary to turn the eye to a position where a single action of each muscle predominates when evaluating the individual muscles. A key principle for muscle testing is: if a muscle has two actions and you perform one of those two, then it can't perform its other action. Superior and inferior recti turn the eye in and up or in and down. Superior and inferior oblique turn the eye out and down or out and up. So, if you turn your eye in (with the superior and inferior rectus as well as medial rectus), then only superior and inferior oblique can move the eye down or up (because the superior and inferior recti are already shortened by turning the eye in - they can't shorten any more). Similarly, if you turn the gaze out (with the obliques and lateral rectus) then only superior and inferior rectus can turn the eye up or down. In this case, the patient has the eye turned inward, so the doctor must be testing the oblique muscles. The superior oblique muscle is the muscle that lowers the eye when it is turned inward. Since the patient can't do this, the superior oblique must not be functioning, and this muscle is innervated by the trochlear nerve. Abducens (CN VI) innervates the lateral rectus muscle, which is not involved in the eye test. The nasociliary nerve comes from the ophthalmic division of the trigeminal nerve (V1). It is a sensory nerve to the eyeball that also carries some sympathetic fibers. The inferior division of the oculomotor nerve innervates inferior rectus, inferior oblique, and medial rectus. All of these muscles appear to be functioning. Finally, the superior division of the oculomotor nerve innervates levator palpebrae superioris and superior rectus. These are not the muscles that appear to be malfunctioning.

The layer of encircling cervical fascia that splits to enclose sternocleidomastoid and trapezius muscles is the: Superficial fascia Superficial layer of deep fascia Infrahyoid fascia Prevertebral fascia Visceral fascia

uperficial layer of deep fascia The superficial layer of deep cervical fascia extends between trapezius and sternocleidomastoid in the posterior triangle and between the paired sternocleidomastoid muscles in the anterior triangle. Superficial fascia is the same as subcutaneous tissue--the tissue that you see as you remove the skin. Infrahyoid fascia is the layer of deep fascia that covers the strap muscles in a sheet. Prevertebral fascia is the deep fascia surrounding the vertebral column and the associated muscles (like the scalenes). Finally, the visceral fascia is the fascia that encloses the deep structures of the neck, including the thyroid, the larynx and trachea, and the esophagus. The anterior part of that fascia is the pretracheal fascia, and the posterior part of the visceral fascia is the buccopharyngeal fascia.

The middle meningeal artery: enters the skull through the foramen ovale passes through a split in the trunk of the mandibular nerve (V3) is typically a branch of the second part of the maxillary artery supplies blood to the temporal lobe of the brain usually arises deep to the neck of the mandible

usually arises deep to the neck of the mandible The middle meningeal artery is always found deep to the lateral pterygoid muscle, so it is arising deep to the neck of the mandible. It enters the skull by passing through the foramen spinosum, not through foramen ovale. Remember--foramen ovale transmits the mandibular branch of trigeminal. Although the middle meningeal artery appears to pass through a fork in the auriculotemporal nerve, it does not pass through a split in the trunk of V3. The middle meningeal artery is not a branch of the second part of the maxillary artery--it is a very early branch that separates from the maxillary artery almost immediately. It supplies blood to the dura mater and the bones of the cranial vault, but it does not supply the temporal lobe of the brain. The temporal lobe is supplied by the middle cerebral artery.


Kaugnay na mga set ng pag-aaral

Business Management Final Review

View Set

Consumer Protection - Chapter 24

View Set

Operations with Scientific Notation

View Set

How was race defined and redefined in Jeffersonian America? How did people of color respond?: Free and Enslaved Black Americans and the Challenge to Slavery/Native American Power and the United States

View Set

MRKT 341 Ch. 10 Marketing Channels Quiz (UNL)

View Set